Módulo 7 Med. Interna Flashcards

1
Q

A 55-year-old man with a history of chronic alcohol dependence presents with fever and cough productive of mucopurulent sputum for 2 days. His temperature is 39.0 C (102.0 F), blood pressure is 120/75 mm Hg, pulse is 110/min, and respirations are 26/min. Auscultation of the chest reveals rales and decreased breath sounds in the left lower lung field. Chest x-ray films show a pulmonary infiltrate in the lower left lobe. Which of the following is the most likely pathogen?

(A) Influenza virus
(B) Klebsiella pneumoniae
(C) Legionella pneumophila
(D) Mycoplasma pneumoniae
(E) Pneumocystis carinii
(F) Staphylococcus aureus

A

Respuesta: B

The correct answer is B. In this case, productive cough of recent onset, fever, auscultatory findings, and radiologic evidence of a pulmonary infiltrate are consistent with acute bronchopneumonia. Klebsiella pneumoniae is the most common organism causing community-acquired pneumonia in chronic alcoholics.

Influenza virus (choice A) may result in respiratory symptoms mimicking pneumonia, but chest x-ray films do not reveal lobar pulmonary infiltrates, unless there is a superimposed bacterial infection.

Legionella pneumophila (choice C) is one of the most common causes of community-acquired pneumonia. It usually affects individuals with some degree of immune impairment or respiratory damage, especially heavy smokers and patients with chronic obstructive pulmonary disease (COPD). Outbreaks of legionellosis result from exposure to contaminated sources, such as air conditioning towers or shower heads.

M. pneumoniae (choice D) is the most frequent etiologic agent of atypical pneumonia, characterized by dry cough, low-grade fever, and relatively few localizing symptoms. Small outbreaks within close communities are characteristic of pneumonia due to this microorganism.

Pneumocystis carinii (choice E) causes pneumonia in severely immunocompromised patients. It is the most common presenting manifestation of AIDS patients. The clinical picture consists of high fever and severe respiratory compromise, which often progresses to acute respiratory failure.

Staphylococcus aureus (choice F) causes pneumonia most often in hospitalized patients or as a complication of influenza. It is characterized by necrotizing inflammation and frequent development of cavitation.

How well did you know this?
1
Not at all
2
3
4
5
Perfectly
2
Q

A 45-year-old man presents to the emergency department, complaining of the sudden onset of left substernal chest pain. He describes this as a chest pressure that rates 8 on a scale of 1 to 10. He has never had these symptoms before. He has a history of hypercholesterolemia, for which he takes a lipid lowering drug. On physical examination, he appears anxious and is diaphoretic. His blood pressure is 100/90 mm Hg, and his pulse is 110/min. The remainder of the physical examination is unremarkable. He undergoes an ECG, which reveals ST elevation in leads II, III, and aVF. He is diagnosed with an acute myocardial infarction. On cardiac catheterization, which of the following vessels will most likely be occluded?

(A) Circumflex artery
(B) Left anterior descending artery
(C) Left coronary artery
(D) Posterior descending artery
(E) Right coronary artery

A

Respuesta: E

The correct answer is E. The right coronary artery supplies the inferior and posterior segments of the heart. ST changes would be seen in II, III, and aVF.

The circumflex artery (choice A) supplies the anterolateral regions of the heart and would affect leads V5-V6 and I-aVL.

The left anterior descending (LAD) artery (choice B) supplies the anteroseptal and anteroapical regions, and a lesion would be represented by ST changes in V1-V4.

Left coronary artery occlusion (choice C) would encompass regions supplied by the LAD and circumflex and would affect V1-V6 and I, and aVL.

The posterior descending artery (choice D) supplies the posterior aspect of the heart and would show tall R waves in V1-V2.

How well did you know this?
1
Not at all
2
3
4
5
Perfectly
3
Q

A 37-year-old woman presents to the emergency department with emesis. She states that she has fibromyalgia syndrome and uses a number of “pain-killers” to control her pain. On waking this morning, she promptly vomited “coffee grounds.” On examination, she is cool but well perfused. Her blood pressure is 120/70 mm Hg, and her pulse is 110/min, with no orthostasis. The remainder of her physical examination is unremarkable. A nasogastric tube is passed, which returns 200 mL of coffee ground material that eventually clears with normal saline lavage. The patient is sent for endoscopy. Which of the following is the most likely diagnosis?

(A) Esophagitis
(B) Esophageal varices
(C) Gastric neoplasm
(D) Gastric ulcers
(E) Mallory-Weiss tears

A

Respuesta: D

The correct answer is D. The most common causes of upper gastrointestinal (UGI) bleeds are peptic ulcer disease (PUD, 45 to 50%), gastritis (30%), varices (10%), and then the remainder of causes such as Mallory-Weiss tears, esophagitis, and neoplasms. In any patient with a history of “pain-killer” use, especially women with rheumatologic conditions, the diagnosis of gastritis or gastric ulcers secondary to NSAID use must be suspected. The most common cause of these two conditions is NSAID use.

Esophagitis (choice A) is usually due to acid reflux disease and is not a significant cause of UGI bleeding.

Esophageal varices (choice B) are a very common cause of UGI bleeds in patients with cirrhosis. In the U.S., the most common causes of cirrhosis are alcohol and hepatitis virus infection. Worldwide, schistosomiasis is the most common cause. Since this patient has none of the above diseases, the likelihood of her having varices is almost zero.

Gastric neoplasm (choice C), although accounting for a small percentage of patients with UGI bleeding, requires other associated findings of cancer to be suspected. Gastric cancer in particular is associated with early satiety, epigastric pain, a palpable abdominal mass, and certain nitrate-containing foods.

Mallory-Weiss tears (choice E) are small esophageal tears induced by acute acid erosion secondary to vomiting. It should be suspected in patients who have the triad of hematemesis, alcohol abuse, and vomiting. It is not a cause of severe, prolonged, or recurrent UGI bleeding

How well did you know this?
1
Not at all
2
3
4
5
Perfectly
4
Q

Twenty-five guests at an outdoor wedding party are abruptly stricken with severe symptoms, including cramping abdominal pain, nausea, vomiting, and nonbloody diarrhea. Approximately 3 hours earlier, they had eaten refreshments that had been sitting in the sun for several hours, including pastries filled with whipped cream. Within 12 hours, everyone is feeling much better. Which of the following was the most likely cause of the abdominal distress?

(A) Campylobacter
(B) Clostridium botulinum
(C) Clostridium perfringens
(D) Escherichia coli O157:H7
(E) Staphylococcus aureus

A

Respuesta: E

The correct answer is E. The toxin of coagulase positive Staphylococcus characteristically produces food poisoning with abrupt onset of severe gastrointestinal symptoms 2-8 hours after ingestion of contaminated custards, cream-filled pastry, milk, processed meat, or fish. The diarrhea is characteristically nonbloody because the toxin does not produce mucosal ulceration. Most patients recover spontaneously within 12 hours; a few develop sufficiently severe acid-base and electrolyte imbalance to cause shock and even death, particularly among infants, the elderly, and the chronically ill.

Campylobacter (choice A) is the most common bacterial cause of infectious diarrhea in the U.S., typically producing illness lasting several days.

Clostridium botulinum toxin (choice B) produces botulism, characterized by a flaccid paralysis.

Clostridium perfringens (choice C) causes usually relatively mild gastroenteritis 6-24 hours after ingestion of contaminated food, notably meats.

Escherichia coli O157:H7 (choice D) causes acute bloody diarrhea due to a toxin similar to that of Shigella.

How well did you know this?
1
Not at all
2
3
4
5
Perfectly
5
Q

A 55-year-old man presents with a 2-day history of hemoptysis. He reports an acute onset of eight episodes of coughing bright blood. He has coughed a teaspoon worth of blood on average in each instance. He reports no other symptoms, except for a cough productive of 5-10 mL of sputum each morning. He has a history of chronic obstructive pulmonary disease, for which he takes bronchodilators. He has smoked 30 cigarettes daily for the past 30 years. Physical examination is normal, and an x-ray film is clear. Which of the following is the most likely cause of this man’s hemoptysis?

(A) Alpha-1 antitrypsin deficiency
(B) Bronchiectasis
(C) Bronchogenic carcinoma
(D) Chronic bronchitis
(E) Pulmonary tuberculosis

A

Respuesta: D

The correct answer is D. This patient most likely has chronic bronchitis, as evidenced by his cough and sputum production. Bronchitis involves excessive mucus production in the bronchial tree, leading to a productive cough for at least 3 months during each of 2 successive years. It is among the most common causes of hemoptysis in adults. However, lung cancer must be high on the differential diagnosis list.

Alpha-1-antitrypsin deficiency (choice A) is a genetic factor predisposing to emphysema. This should not cause hemoptysis. It would certainly lead to wheezing and symptoms seen in chronic obstructive pulmonary disease.

A normal chest x-ray film does not necessarily rule out bronchiectasis (choice B). Bronchiectasis is a pathologic, irreversible dilatation of the bronchi that is caused by destruction of the bronchial wall, usually resulting from suppurative infection of an obstructed bronchus. Symptoms may occasionally include hemoptysis.

Bronchogenic carcinoma (choice C) must be considered in someone with a long smoking history and hemoptysis. Statistically, bronchitis is still more common. Chest x-ray films can be clear in lung cancer; if there is clinical suspicion, this patient must get a chest CT to rule out a mass.

Chest x-ray films can be clear in a patient with pulmonary tuberculosis (choice E). Radiographic signs, if present, include apical granulomas on lung chest x-ray. Other symptoms may include cough, weight loss, and hemoptysis. Acid fast bacilli (AFB) smear and cultures may be positive for Mycobacterium tuberculosis. Pulmonary tuberculosis is less likely than chronic bronchitis in this smoker.

How well did you know this?
1
Not at all
2
3
4
5
Perfectly
6
Q

A 30-year-old man is recovering from abdominal trauma and now develops multiple bruises all over his body. He has completed a 2- week course of antibiotics and has also been receiving nutrition parenterally for 21 days. He is otherwise healthy and has never been on medications prior to this admission. On physical examination, his vital signs are stable. He has multiple ecchymoses on his abdomen. The heart is regular in rate and rhythm. His prothrombin time is elevated. His hematocrit and platelet count are normal. Which of the following is the most likely cause of his easy bruising?

(A) Calcium deficiency
(B) Disseminated intravascular coagulation (DIC)
(C) Magnesium deficiency
(D) Sepsis
(E) Vitamin K deficiency

A

Respuesta: E

The correct answer is E. Antibiotics can suppress normal gut flora that produce vitamin K. In addition, most total parenteral nutrition (TPN) preparations do not include vitamin K. This vitamin is essentially in the normal functioning of coagulation factors II, VII, IX, and X; deficiency frequently leads to coagulation defects, manifested by easy bruising and prolonged prothrombin time (PT).

Calcium is a cofactor in the coagulation pathway, but deficiency is generally not associated with easy bruisability (choice A).

Disseminated intravascular coagulation (DIC) (choice B) can lead to a coagulopathy as a result of the consumption of coagulation factors. However, the patient would also have reduced platelets.

Magnesium deficiency (choice C) can lead to cardiac arrhythmias but not a bleeding diathesis.

Sepsis (choice D) can result in DIC and generalized shock. The patient, however, would be pressor dependent and have elevated PT and partial thromboplastin time (PTT) and reduced platelets

How well did you know this?
1
Not at all
2
3
4
5
Perfectly
7
Q

A 27-year-old woman comes to her physician because of weakness, weight loss, and amenorrhea for 6 months. Her blood pressure is 100/65 mm Hg. On examination, increased skin pigmentation is seen, especially around the nipples and over the knees, elbows, and knuckles. Laboratory analysis shows:

  • Sodium 125 mEq/L
  • Potassium 6.3 mEq/L
  • Chloride 100 mEq/L
  • Calcium 10 mEq/L

Complete blood count shows mild lymphocytosis with eosinophilia. Low plasma levels of cortisol and high levels of ACTH are detected on a blood sample drawn at 8 am. Which of the following is the most common cause of this disease in the U.S.?

(A) Adrenoleukodystrophy
(B) Autoimmune destruction
(C) Bilateral adrenal hemorrhage
(D) Fungal infection
(E) Metastatic disease
(F) Tuberculosis

A

Respuesta: B

The correct answer is B. This patient manifests a classic syndrome of hypocortisolism, the most common manifestations of which include chronic weakness, menstrual disturbances, skin melanosis, hypotension, hyponatremia, and hyperkalemia. Eosinophilia and relative lymphocytosis are also frequent findings. The most frequent cause of Addison disease (the designation given to this clinical picture) in the U.S. is autoimmune destruction of the adrenal glands, evidenced by lymphocytic infiltration and progressive atrophy of the adrenal cortex. High ACTH levels represent a compensatory response of the anterior pituitary gland. High ACTH levels, skin hyperpigmentation, and hyperkalemia are absent in cases secondary to hypopituitarism. The disease is more frequent in women.

Adrenoleukodystrophy (choice A) is a rare X-linked hereditary disorder resulting in accumulation of very long chain fatty acids in the adrenals, testes, and CNS. Thus, adrenal insufficiency, neurologic deficits, and hypogonadism constitute the predominant manifestations. This condition accounts for one third of cases of adrenal insufficiency in male children.

Bilateral adrenal hemorrhage (choice C) may cause adrenal insufficiency and occurs as a complication of anticoagulation therapy, major traumas, or open heart surgery.

Fungal infection (choice D) was a very rare cause of adrenal insufficiency until the AIDS epidemic. Disseminated histoplasmosis is probably the most common fungal infection causing adrenal insufficiency in AIDS patients. Coccidioidomycosis and CMV infections are other possible etiologies of Addison disease in immunocompromised patients.

Metastatic disease (choice E) may result in adrenal insufficiency if both glands are affected. Metastatic carcinomas have a peculiar propensity to involve the adrenal gland despite the small size of this organ.

Tuberculosis (choice F) was once the most common cause of adrenal insufficiency, and is still the most common cause in areas in which tuberculosis is endemic.

How well did you know this?
1
Not at all
2
3
4
5
Perfectly
8
Q

A 46-year-old woman complains of dyspnea on exertion and orthopnea that started 7 months ago. She has a prior history of pericarditis, which had been treated with indomethacin. An ECG shows low voltage in the limb leads. A chest radiograph reveals pericardial calcification, and echocardiography shows pericardial thickening. Cardiac catheterization reveals equal pressures in the four cardiac chambers during diastole with all pressures elevated. Which of the following findings on her physical examination would be consistent with a diagnosis of constrictive pericarditis?

(A) Increased neck vein distention on inspiration
(B) Exaggerated first and second heart sounds (S1 and S2)
(C) Extra third heart sound (S3)
(D) Extra fourth heart sound (S4)
(E) Predominance of left-sided symptoms over rightsided symptoms

A

Respuesta: A

The correct answer is A. Constrictive pericarditis is diffuse thickening of the pericardium in reaction to prior inflammation, resulting in reduced distensibility of the cardiac chambers. Cardiac output is limited, and filling pressures are increased to match the external constrictive force placed on the heart by the pericardium. In both constrictive pericarditis and cardiac tamponade, the diastolic pressures are equal in all four chambers of the heart. Jugular veins are distended, indicating systemic venous hypertension. This neck vein distention increases with inspiration and is called Kussmaul’s sign.

The first and second heart sounds (S1 and S2) are reduced in intensity because of reduced sound transmission through the thickened pericardium (choice B).

Patients with congestive heart failure have an extra, third sound (choice C). This occurs during rapid filling of the left ventricle.

The fourth heart sound (choice D) is heard in patients in sinus rhythm and with heart failure. In elderly patients, it may indicate reduced compliance of the stiff ventricle.

In most cases of constrictive pericarditis, the clinical findings of right-sided failure are more prominent than those of left-sided failure (choice E). Thus ascites, jaundice, and edema will be commonly seen.

How well did you know this?
1
Not at all
2
3
4
5
Perfectly
9
Q

A healthy 29-year-old woman comes to the physician for a health maintenance examination. Palpation of the neck reveals a firm, 0.5- cm nodule in the right thyroid lobe. The remainder of the gland is normal. The physical examination is otherwise unremarkable. The patient denies any symptoms attributable to hyperthyroidism. The results of thyroxine and TSH immunoassays are within normal limits. Which of the following is the most appropriate next step in diagnosis?

(A) MRI scan of the neck
(B) CT scan of the neck
(C) Radioactive iodine scan
(D) Fine needle aspiration
(E) Excision

A

Respuesta: D

The correct answer is D. An isolated thyroid nodule is a frequent finding in asymptomatic adults, and most of such are benign. Fine needle aspiration allows a diagnosis in most cases. The material aspirated with a needle is smeared on a slide and stained. In only 15% of cases is the aspirated material “non-diagnostic.” Suspicious cases are followed with repeated fine needle aspiration. Malignant nodules are usually large (>3 cm) and/or fixed to the surrounding parenchyma. Papillary carcinoma is the most common malignant thyroid neoplasm.

Ultrasonography may also be of value in distinguishing solid from cystic nodules and is preferred to MRI scan (choice A) or CT scan (choice B) because of its high sensitivity and lower cost. However, CT and MRI are valuable in defining the extent of malignant tumor, once the diagnosis is made.

Radioactive iodine scan (choice C) is needed when a solitary thyroid nodule is associated with symptoms of thyrotoxicosis. Radioactive iodine scan helps to distinguish a toxic adenoma from Graves disease, in which high uptake is seen in the whole gland.

Excision of a thyroid nodule (choice E) is performed if it proves to be malignant, or in case of a hot (i.e., hyperfunctioning) nodule causing thyrotoxicosis.

How well did you know this?
1
Not at all
2
3
4
5
Perfectly
10
Q

A 40-year-old IV drug addict complains of right-sided weakness and headache over the past week. He has been previously healthy and is on no prescribed medications. On physical examination, he is afebrile, cachectic, and in mild distress. His neck is supple, and his lungs and skin are clear. Cardiac examination reveals no murmur. He has a mild right hemiparesis. He is tested for HIV and found to be negative. An echocardiogram reveals no valve vegetation. Which of the following is the most likely diagnosis?

(A) Bacterial endocarditis
(B) Bacterial meningitis
(C) Brain abscess
(D) Cryptococcal meningitis
(E) Foreign body embolus

A

Respuesta: C

The correct answer is C. IV drug users are prone to developing bacteremia, which can lead to a brain abscess. Patients with brain abscesses are typically afebrile and can exhibit progressive neurologic dysfunction.

IV drug abusers are prone to developing bacterial endocarditis (choice A), but they are typically febrile. This patient’s echocardiogram, and physical examination lead away from the diagnosis. However, in the management of this patient, blood cultures should be drawn, and suspicion of endocarditis must be high.

Patients with bacterial meningitis (choice B) are typically toxic appearing and febrile and have positive signs of meningeal irritation. They also often exhibit nuchal rigidity. Elevated WBC in the CSF, decreased glucose level, elevated protein, and a predominance of neutrophils are indicative of this diagnosis.

HIV meningitis causes a headache and meningeal irritation. Focal neurologic deficits do not occur. Cryptococcal meningitis (choice D) typically presents with altered behavior and a headache. Stroke-like events are rare.

IV drug use can lead to a foreign body embolus (choice E) and apoplectic neurologic problems. An embolus may reach the brain via a right-to-left cardiac shunt or pulmonary arteriovenous malformation if the injection is venous. Embolic phenomena are more common in the setting of endocarditis.

How well did you know this?
1
Not at all
2
3
4
5
Perfectly
11
Q

A 75-year-old woman is brought to the emergency department after being found unconscious by a neighbor. The woman has a history of type 2 diabetes. A stat blood draw demonstrates a plasma glucose of 975 mg/dL. Which of the following additional findings would be most consistent with the patient’s probable diagnosis?

(A) Blood urea nitrogen 5 mg/dL
(B) Plasma strongly positive for ketones
(C) Serum creatinine 0.3 mg/dL
(D) Serum osmolality 380 mOsmol/kg
(E) Serum sodium 132 mEq/L

A

Respuesta: D

The correct answer is D. This woman is in a nonketotic hyperglycemic hyperosmolar coma, a feared complication of type 2 diabetes mellitus that is associated with a 50% mortality rate. The basic problem is when extreme hyperglycemia occurs, glucose spills into the urine and can cause profound dehydration since the glucose acts as an osmotic diuretic. Features of this syndrome include CNS alterations, extreme hyperglycemia (typical values in the range of 1000 mg/dL), and dehydration. These features lead to hyperosmolality (the correct choice in this case; normal values are less than about 290 mOsmol/kg), mild metabolic acidosis, no ketonemia to minimal hyperketonemia, and prerenal azotemia. Diabetic ketoacidosis, the other diagnosis that should be considered, is uncommon in type 2 diabetics and is associated with lower blood glucose levels than nonketotic hyperglycemic hyperosmolar coma.

Blood urea nitrogen (choice A) and serum creatinine (choice C) are usually elevated rather than decreased in nonketotic hyperglycemic hyperosmolar coma.

Strongly positive ketones in blood (choice B) are a feature of diabetic ketoacidosis. Minimally elevated ketones in the blood are sometimes seen in nonketotic hyperglycemic hyperosmolar coma.

Serum sodium (choice E) is usually normal to increased in nonketotic hyperglycemic hyperosmolar coma.

How well did you know this?
1
Not at all
2
3
4
5
Perfectly
12
Q

An elderly woman complains to her physician of chronic constipation. Her physician performs a full physical examination, including a rectal examination to exclude masses. A complete blood count, thyroid-stimulating hormone, fasting glucose, and electrolyte studies are also ordered. Neither the physical examination nor the laboratory tests reveal any abnormalities that might suggest serious disease. Which of the following is the most appropriate next step in management?

(A) Docusate
(B) Lactulose
(C) Magnesium phosphate
(D) Mineral oil
(E) Psyllium

A

Respuesta: E

The correct answer is E. Chronic constipation is a common problem in the elderly; however, before reassuring the patient, the physician has an obligation to exclude serious disease such as colon cancer. Once the physician is reasonably convinced that there is no serious underlying pathology, the next steps are to suggest increasing fiber in the diet and to discontinue any medications that may be causing the constipation. If these steps fail, then the addition of bulking agents (bran, psyllium, calcium polycarbophil, or methylcellulose) is warranted. Long-term use of other types of laxatives is not recommended.

Docusate (choice A) is a wetting agent (detergent laxative) that softens stools by increasing their water content.

Lactulose (choice B) and magnesium phosphate (choice C) are osmotic agents sometimes used to prepare patients for diagnostic bowel procedures.

Mineral oil (choice D) softens fecal matter but is not recommended for long-term use because it may decrease absorption of fat soluble vitamins.

How well did you know this?
1
Not at all
2
3
4
5
Perfectly
13
Q

A 22-year-old woman is seen by a physician because she feels poorly. Physical examination demonstrates waxy pallor of her skin and mucous membranes. She also has multiple purpura on her extremities that she attributes to minor trauma, such as hitting her hand accidentally on a drawer. Blood studies are performed, demonstrating a red cell count of 1.5 million/μL, white count of 1300/μL (80% lymphocytes), and platelet count of 40,000/μL. Reticulocytes are absent. All blood cells seen have normal morphology. Bone marrow biopsies obtained from the hips bilaterally show predominately fat, with markedly diminished precursors in all blood cell lines. Which of the following is the most likely diagnosis?

(A) Aplastic anemia
(B) Iron deficiency anemia
(C) Folate deficiency anemia
(D) Myelophthisic anemia
(E) Vitamin B12 deficiency anemia

A

Respuesta: A

The correct answer is A. This patient has aplastic anemia. A characteristic feature of this condition is that the growth of erythrocyte, granulocyte, and megakaryocyte precursors is markedly impaired. The marrow is usually replaced by adipose tissue. Aplastic anemia typically develops insidiously, but may have a more rapid course. In about half of cases, no cause is ever identified. In the remainder, causes may include chemical exposures (e.g., benzene and inorganic arsenic), radiation, or drug reactions (e.g., antineoplastic agents, antibiotics, anticonvulsants, and NSAIDs), and parvovirus B19 in patients with hemoglobinopathies or spherocytosis. Historically, the condition has required marrow transplantation, but this therapy is now reserved for patients who fail to improve with equine antithymocyte globulin or cyclosporine therapy.

Iron deficiency anemia (choice B) produces a microcytic anemia.

Folate deficiency anemia (choice C) and vitamin B12 deficiency anemia (choice E) produce a megaloblastic anemia.

Myelophthisic anemia (choice D) can clinically resemble aplastic anemia, but bone marrow studies would demonstrate tumor, granulomatous disease, or fibrosis replacing the normal marrow.

How well did you know this?
1
Not at all
2
3
4
5
Perfectly
14
Q

An 8-year-old girl is brought to the physician’s office by her parents because of 2 days of progressive left-sided facial weakness. On physical examination, a left facial droop is noted. Vesicular eruptions are seen in the left external auditory canal, as well as on the left side of the pharynx. Which of the following is the most likely diagnosis?

(A) Bell palsy
(B) Guillain-Barré syndrome
(C) Horner syndrome
(D) Meniere disease
(E) Ramsay Hunt syndrome

A

Respuesta: E

The correct answer is E. This question asks you to differentiate between different types of peripheral motor neuropathies. Ramsay Hunt syndrome, caused by herpes zoster infection of the geniculate ganglion, results in facial palsy. It differs from the other neuropathies in that there is usually a vesicular eruption, typical of herpes infections.

Bell palsy (choice A), the most common form of facial paralysis, is idiopathic. The onset is abrupt, with maximal weakness in the first 48 hours. Eighty percent of patients fully recover in a few weeks.

Guillain-Barré syndrome (choice B) is an acute inflammatory polyradiculoneuropathy, causing bilateral facial palsy and usually producing areflexic motor paralysis. A viral illness often precedes the onset of neuropathy.

Horner syndrome (choice C) affects the oculosympathetic nerves, usually ipsilaterally, specifically causing unilateral miosis and ptosis, with normal pupillary response to light. Hemianhidrosis of the face also occurs.

Meniere disease (choice D) manifests with recurrent vertigo and is associated with tinnitus and progressive deafness. There is facial paralysis.

How well did you know this?
1
Not at all
2
3
4
5
Perfectly
15
Q

A 25-year-old man consults a dermatologist because of a rash. He states that the rash started with a single lesion on his chest, which grew larger; other lesions then developed. He thinks he might have ringworm. Physical examination demonstrates multiple scaly lesions on his chest and back. The largest of these, which the patient says was his first, is 5 cm in diameter, oval, and rose colored. A slightly raised border (collarette) is seen around the edge of the lesion. Many small plaques, about 1 cm in diameter with a similar appearance, are also seen. Which of the following is the most likely diagnosis?

(A) Molluscum contagiosum
(B) Pityriasis rosea
(C) Pityriasis rubra pilaria
(D) Rosacea
(E) Scabies

A

Respupesta: B

The correct answer is B. The patient has pityriasis rosea, and the large lesion is known as the “herald patch.” Any description of this disease in a question will probably either use this term or describe the initial, larger lesion. The scaly lesions tend to involve the trunk and may be either oval or circinate (increasing an initial impression of ringworm).The lesions on the back may follow the lines of cleavage of the skin, producing a “Christmas tree” appearance of the lesion distribution. The condition is self-limited, but may persist more than 2 months. It is suspected to be infectious in nature, with potential causative species including a picornavirus, herpes virus 7, and Mycoplasma.

Molluscum contagiosum (choice A) causes multiple small papules with umbilicated centers.

Pityriasis rubra pilaria (choice C) has a predilection for involving the hands and soles and does not produce a herald patch.

Rosacea (choice D) causes telangiectasia, erythema, papules, and pustules of the nose and cheeks.

Scabies (choice E) would be suggested in a question stem if the physician found small skin burrows in addition to papules. Also, itching is prominent with scabies.

How well did you know this?
1
Not at all
2
3
4
5
Perfectly
16
Q

Four hours after repairing a dissected aortic aneurysm, a patient develops paraplegia below the T10 level. He is nonresponsive to pain or temperature but has preserved proprioception. Which of the following arteries is most likely affected?

(A) Anterior cerebral
(B) Middle cerebral
(C) Posterior cerebral
(D) Thalamostriate
(E) Ventral spinal

A

Respuesta: E

The correct answer is E. This patient has a bilateral loss of pain and temperature sensation, with preserved proprioception, below the T10 dermatome. This implies damage at the spinal, rather than the brainstem, level. The ventral (anterior) spinal artery’s course begins in the anterior median sulcus of the spinal cord, with branches supplying the ventral and lateral funiculi and most of the spinal cord gray matter. Damage to the ventral spinal artery produces bilateral loss of pain and temperature sensation below the level of the lesion as a result of injury to the spinothalamic tracts on both sides.

Damage to the anterior cerebral artery (choice A) would be expected to produce paresis of the contralateral lower extremity resulting from injury to the paracentral lobule.

Damage to the middle cerebral artery (choice B) would be expected to result in some degree of contralateral hemiparalysis, primarily of the face and upper extremity. If the dominant hemisphere were affected, aphasia would be expected as well.

Damage to the posterior cerebral artery (choice C) would diminish blood flow through the calcarine artery, resulting in contralateral homonymous hemianopia. Lack of blood flow through the thalamic branches of the posterior cerebral artery would produce hemiplegia with varying degrees of contralateral sensory loss. Thalamic pain (constant excruciating pain in the hemiplegic extremities) might appear later.

The thalamostriate arteries (choice D) are small branches of the middle cerebral arteries that supply the internal capsule and portions of the basal ganglia. They are particularly susceptible to rupture and hemorrhage in patients with hypertension or arteriosclerosis. In such cases, complete contralateral paralysis usually ensues.

How well did you know this?
1
Not at all
2
3
4
5
Perfectly
17
Q

A 24-year-old woman comes to the emergency department because of abdominal pain, nausea, and anorexia for 24 hours. She is sexually active but does not take oral contraceptives. Her last menstrual period was 2 weeks ago. Her temperature is 38.0 C (100.4 F). The pain is constant and localized in the right lower abdomen, where palpation elicits guarding and rebound tenderness. Pain in the right lower quadrant of the abdomen is also provoked by palpation of the left lower quadrant. Bowel sounds are absent. Pelvic examination is normal. Laboratory investigations show moderate neutrophilic leukocytosis and β-hCG within normal limits. Urinalysis shows 2 erythrocytes per high power field. Which of the following is the most appropriate next step in management?

(A) Antibiotic therapy
(B) Barium enema
(C) Ultrasonography of urinary tract
(D) Dilatation and curettage
(E) Appendectomy

A

Respuesta: E

The correct answer is E. This patient has the classic symptomatology of acute appendicitis. Pain in the right lower quadrant provoked by palpation of other areas of the abdominal wall is known as Rovsing sign. Presence of rebound tenderness is a sign of early peritoneal involvement. The pain usually begins in the periumbilical region and then moves to its characteristic location. Anorexia is the second most frequent symptom and usually precedes or accompanies pain. Vomiting is often present and usually follows the onset of pain. Fever, if present, is usually modest. Moderate neutrophilic leukocytosis is often detected. A few RBCs may be seen on urinalysis because of secondary involvement of the adjacent ureter. Pelvic exam should be performed in all women with acute abdominal pain to rule out intrauterine or ectopic pregnancy, pelvic inflammatory disease, ovarian cysts, or malignancies. A normal pelvic examination and negative assay for β-hCG makes pregnancy a highly unlikely cause of this patient’s symptomatology. In the presence of this classic picture, laparotomy and appendectomy are mandatory, but surgery is often performed when less than typical symptomatology is observed. It is, in fact, preferable to remove a few normal appendices than to leave untreated a potentially lifethreatening condition.

Antibiotic therapy (choice A) is inadequate treatment for appendicitis. Antibiotics, however, are used pre- and postoperatively to decrease the incidence of wound infections.

No radiologic investigation can confirm a clinical diagnosis of appendicitis. Barium enema (choice B) may be helpful in uncertain cases, but is often unnecessary. A filling defect in the cecum is highly suggestive of appendicitis.

Ultrasonography of urinary tract (choice C) is warranted in cases of abdominal pain attributable to urolithiasis or other urologic conditions. As mentioned, the presence of few erythrocytes in the urine is frequently seen in association with appendicitis and does not indicate urinary tract disease.

Dilatation and curettage (choice D) would not be appropriate in a woman with suspected appendicitis.

How well did you know this?
1
Not at all
2
3
4
5
Perfectly
18
Q

A 33-year-old woman complains of diplopia in the early evening every day for the past month, which resolves following sleep. She also complains of jaw weakness after eating large meals. She has no significant past medical history. Physical examination is normal, except repeated blinking elicits a ptosis that resolves following the administration of IV edrophonium. Which of the following is the most likely diagnosis?

(A) Botulism
(B) Eaton-Lambert syndrome
(C) Guillain-Barré syndrome
(D) Multiple sclerosis
(E) Myasthenia gravis

A

Respuesta: E

The correct answer is E. Myasthenia gravis is an autoimmune disease caused by circulating antibodies that bind to acetylcholine receptors on the postsynaptic membrane. The disease is characterized by weakness and fatigue. The weakness usually begins in the extraocular muscles with ptosis and diplopia. The symptoms may be localized to the ocular muscles or generalized. The weakness usually becomes more prominent toward the end of the day or following continuous use of affected muscles. Other bulbar muscles may be involved, causing difficulty in swallowing, chewing, or speaking. Closing the eyes or relaxing makes the symptoms disappear. Ocular weakness can be induced with repetitive blinking, with the patient developing ptosis. Administration of edrophonium causes a transient resolution of symptoms.

Botulism (choice A) is caused by the exotoxin of Clostridium botulinum and occurs following the ingestion of contaminated food. The toxin interferes with the release of acetylcholine at the neuromuscular junction. Symptoms usually appear several days after ingestion and include blurry vision, diplopia, and difficulty swallowing. Gastrointestinal symptoms may develop, and the weakness spreads rapidly to cause paralysis of limb, cranial, and respiratory muscles. Administration of edrophonium will not reverse symptoms.

Eaton-Lambert syndrome (choice B) is a presynaptic disorder of the neuromuscular junction. Multiple muscle groups, most commonly the proximal muscles of the lower limbs, are affected. Diplopia and ptosis may also be present. Eaton-Lambert is readily differentiated from myasthenia gravis, as patients with Eaton-Lambert syndrome have markedly depressed or absent reflexes, autonomic changes, and slow incremental responses with repetitive nerve stimulation. Treatment with edrophonium does not improve symptoms.

Guillain-Barré syndrome (choice C) is an acute polyneuropathy characterized by a rapidly progressive, predominantly motor neuropathy that may paralyze all voluntary muscles, including those supplied by cranial nerves. Pain is common, along with some degree of autonomic dysfunction. Bilateral facial paralysis is also common and helps to differentiate the syndrome from other polyneuropathies. Onset is usually 2-3 weeks after a respiratory infection. On examination, the weakness usually progresses from the lower to upper extremities, and finally to the face. Deep tendon reflexes are absent, and symmetric weakness of all extremities is noted.

Multiple sclerosis (choice D) is the most common immune demyelinating disorder of the CNS. It usually presents between the ages of 20 and 40 and is characterized by remissions and exacerbations of neurologic dysfunction. It usually involves several different sites of the CNS and progresses over many years. The typical presentation is that of an otherwise healthy woman developing an acute loss of vision, diplopia, vertigo, incontinence, or paralysis. Resolution of symptoms will generally occur over several weeks and do not resolve with rest or edrophonium and cannot be elicited with repetitive use.

How well did you know this?
1
Not at all
2
3
4
5
Perfectly
19
Q

A previously healthy 50-year-old woman presents with progressive muscle pain and weakness for 3 weeks. The symptoms are localized mainly to the proximal muscle groups, affecting the deltoid and the pelvic muscles. On physical examination, there is tenderness on palpation of affected muscles and objective loss of strength. Laboratory investigations reveal a serum creatine kinase level of 2000 U/L. Electromyography demonstrates myopathic changes. A biopsy of the deltoid muscle reveals atrophy and necrosis of scattered myofibers with endomysial inflammatory infiltration mostly composed of lymphocytes. Which of the following is the most likely diagnosis?

(A) Denervation atrophy
(B) Dermatomyositis
(C) Inclusion body myositis
(D) Polymyositis
(E) Systemic lupus erythematosus

A

Respuesta: D

The correct answer is D. Polymyositis presents with the described clinical picture, i.e., proximal muscle weakness and pain. Because of ongoing destruction of myofibers, the levels of creatine kinase are often strikingly elevated. The disease is thought to arise from an abnormal T-lymphocyte-mediated response targeting myofiber antigens. This condition is not associated with underlying malignancies.

Denervation atrophy (choice A) results from loss of innervation of skeletal muscle (e.g., after traumatic nerve transection or axonal degeneration). Inflammation is minimal or absent, while myofibers undergo progressive shrinkage without necrosis. Myopathic changes seen with electromyographic studies, proximal muscle involvement, and histopathologic changes argue against a diagnosis of denervation atrophy.

Dermatomyositis (choice B) may mimic polymyositis on muscle biopsy, but skin involvement is absent in the latter. In dermatomyositis, muscle biopsy often shows characteristic perifascicular atrophy, with the atrophic fibers mainly distributed at the periphery of fascicles. A significant number of cases (up to 30%) are associated with some form of visceral malignancy.

Inclusion body myositis (choice C) has a predilection for the distal musculature and presents histologically with modest degrees of inflammation. Rimmed vacuoles are the specific morphologic changes that allow a pathologic diagnosis on biopsy.

Systemic lupus erythematosus (choice E) may involve skeletal muscles. However, this systemic disease is usually accompanied by multiorgan involvement, especially the skin, kidneys, and serosal membranes. Inflammatory infiltration in the muscle would be more conspicuous around the vessels.

How well did you know this?
1
Not at all
2
3
4
5
Perfectly
20
Q

During a health maintenance examination, an otherwise healthy 50-year-old man has a blood pressure reading of 150/94 mm Hg supine and 145/92 mm Hg standing. Physical examination does not disclose any abnormalities. Which of the following is the most appropriate next step in management?

(A) Evaluate or refer within 1 week
(B) Evaluate or refer within 1 month
(C) Recheck within 2 months
(D) Recheck in 1 year
(E) Recheck in 2 years
(F) Start treatment with diuretics

A

Respuesta: C

The correct answer is C. According to the recommendations issued in 1997 by the Joint National Committee on Detection, Education, and Treatment of High Blood Pressure, normal adult values of blood pressure are systolic <130 and diastolic <85 mm Hg. Stage 1 (mild) hypertension is defined as a systolic pressure in the range of 140 to 159 mm Hg, or a diastolic pressure in the range of 90 to 99 mm Hg. If stage 1 hypertension is detected, the diagnosis should be confirmed within 2 months. Currently, hypertension is diagnosed as elevation of either the systolic or the diastolic value above the normal level.

A systolic pressure >180 mm Hg or a diastolic pressure >110 mm Hg is defined as stage 3 (severe) hypertension. According to the above recommendations, a patient presenting with these values should be evaluated again or referred within 1 week (choice A).

Patients with stage 2 (moderate) hypertension are those who have a systolic pressure of 160 to 179 mm Hg or a diastolic pressure of 100 to 109 mm Hg. The current recommendation is to evaluate or refer within 1 month (choice B).

A systolic pressure of 130 to 139 mm Hg or a diastolic pressure of 85 to 90 mm Hg is defined as “high normal.” In this case, the recommendation is to recheck in 1 year (choice D) to determine whether a condition of hypertension has developed.

Blood pressure should be rechecked within 2 years (choice E) if the values fall within normal limits, i.e., systolic <130 mm Hg and diastolic <85 mm Hg.

Stage 1 hypertension does not necessarily require pharmacologic treatment, such as diuretics (choice F). Nonpharmacologic approaches, such as exercise, weight loss, and changes in diet or lifestyle, may be sufficient.

How well did you know this?
1
Not at all
2
3
4
5
Perfectly
21
Q

Thirty-six hours after surgical removal of a large parathyroid adenoma, a 50-year-old man becomes irritable and develops a tingling sensation around his mouth and in his hands. Facial spasm can be easily triggered by tapping in front of the ear. Laboratory studies show:

Blood, serum:
- Albumin 4.0 g/dL
- Bicarbonate 25 mEq/L
- Calcium 7.1 mg/dL
- Phosphorus 7.5 mg/dL
- Magnesium 1.8 mEq/L

Arterial blood (room air):
- pH 7.40
- PO2 90 mm Hg
- PCO2 42 mm Hg

Which of the following is the most likely cause of this condition?

(A) Atrophy of the remaining parathyroids
(B) Hungry bone syndrome
(C) Hyperventilation syndrome
(D) Magnesium deficiency
(E) Metastatic parathyroid carcinoma

A

Respuesta: A

The correct answer is A. Low calcium and high phosphorus levels, combined with the characteristic manifestations of hypocalcemia (muscle spasms and perioral paresthesias), are characteristic of hypoparathyroidism. This often occurs as a transient postoperative manifestation following removal of a large parathyroid adenoma, which had previously caused atrophy of the remaining normal glands. Hypocalcemia must be treated with IV calcium gluconate soon after surgery, followed by oral calcium and vitamin D administration.

Hungry bone syndrome (choice B) develops days or weeks following resection of a parathyroid adenoma because of avid calcium uptake by a previously demineralized bone. However, since PTH returns to normal levels as the normal glands become functional again, hypocalcemia is not associated with hyperphosphatemia, as in this case. Hungry bone syndrome is the main reason patients require calcium and vitamin D supplementation for months after surgery.

Hyperventilation syndrome (choice C) may produce symptoms mimicking hypocalcemia, such as paresthesias, agitation, and spasms. CO2 levels are decreased, and pH is increased (respiratory alkalosis).

Magnesium deficiency (choice D) may lead to hypocalcemia and hyperphosphatemia mimicking hypoparathyroidism. It is indeed mediated by impairment of PTH secretion and peripheral resistance to PTH action. But the normal levels of magnesium reported above exclude this cause.

Metastatic parathyroid carcinoma (choice E) would be associated with persistent hyperparathyroidism. Parathyroid carcinoma is a rare cause of hyperparathyroidism.

22
Q

A 69-year-old man presents with the chief complaint of difficulty remembering things. His family states that he also has episodes of confusion and disorientation. A complete physical examination is unrevealing. His score on a mini-mental status examination is 18 of 30. Laboratory results show an RPR reactive-titer of 1:4 and a positive fluorescent treponemal antibody absorption (FTA-ABS) test. A CT scan of the brain is unremarkable. Lumbar puncture reveals 3 red cells and 2 white cells/mm3 of CSF. A CSF VDRL is negative. The patient had a previous anaphylactic reaction to penicillin. Which of the following is the most appropriate management for this patient?

(A) Azithromycin
(B) Ceftriaxone
(C) Chloramphenicol
(D) Doxycycline
(E) No treatment is necessary

A

Respuesta: D

The correct answer is D. This patient has late latent syphilis. He has no physical signs of syphilis, with positive serologic tests and a negative lumbar puncture. Treatment is usually with benzathine penicillin G. In a patient who has a penicillin allergy, alternatives include tetracycline or doxycycline.

Azithromycin (choice A) is used for penicillin-allergic patients with primary syphilis.

Ceftriaxone (choice B) and chloramphenicol (choice C) are used in the treatment of late (tertiary) syphilis.

Because the diagnosis of syphilis has been confirmed with a FTAABS, it is important to determine the stage of the disease. As described above, the patient has late latent syphilis and is at risk for developing tertiary syphilis in the future. Therefore, the patient should be treated, which makes choice E an incorrect answer.

23
Q

A 52-year-old man presents with a 6-month history of slowly progressive exertional dyspnea and swelling of his legs. He admits to heavy alcohol abuse for the past 20 years. His blood pressure is 135/84 mm Hg, pulse is 103/min, and respirations are 18/min. Chest examination shows bilateral rales at the lung base and a gallop rhythm with an S3 . The liver margin is palpable 3 cm below the costal margin, and there is mild splenomegaly. Pitting edema is present in the lower extremities. A chest x-ray film reveals dilatation of the veins in the upper pulmonary lobes and cardiac biventricular enlargement. ECG shows low QRS voltage and occasional premature ventricular beats. Blood studies show:

  • Alanine aminotransferase (ALT) 60 U/L
  • Aspartate aminotransferase (AST) 76 U/L
  • Bilirubin, total 1.0 mg/dL
  • Albumin, serum 4.2 g/dL
  • Prothrombin time (PT) 14 sec
  • Ferritin, serum 100 ng/mL

Endocrine studies of pituitary and thyroid functions are within normal limits. Which of the following is the most likely diagnosis?

(A) Alcoholic cirrhosis
(B) Dilated cardiomyopathy
(C) Hemochromatosis
(D) Hypertrophic cardiomyopathy
(E) Restrictive cardiomyopathy

A

Respuesta: B

The correct answer is B. The clinical picture is that of biventricular failure, with evidence of pulmonary congestion and peripheral edema. Hepatomegaly is the result of passive congestion, not the effect of primary liver disease. ALT and AST are mildly elevated, and other liver function tests [bilirubin levels, prothrombin time (PT), and serum albumin] are within normal limits. The enlargement of the heart detected on chest x-ray is indicative of dilated cardiomyopathy, a condition that may result from chronic alcohol abuse or previous myocarditis (among numerous other causes).

Alcoholic cirrhosis (choice A) is the most frequent form of cirrhosis. Liver function tests would be markedly impaired, with reduced serum albumin and prolonged PT. Ascites is usually present, and the liver may not be palpable in the cirrhotic stage.

Hemochromatosis (choice C) may manifest with congestive heart failure because of myocardial involvement by iron accumulation. However, normal serum ferritin values and the absence of other signs of hemochromatosis (e.g., skin hyperpigmentation, cirrhosis, and pancreatic dysfunction) exclude this hypothesis.

Hypertrophic cardiomyopathy (choice D) does not result in cardiomegaly. Indeed, the heart size may be within normal limits on chest x-ray. The most important finding is thickening of the interventricular septum on echocardiography, which results in stenosis of the outflow tract and a systolic murmur.

Restrictive cardiomyopathy (choice E) is clinically similar to hypertrophic cardiomyopathy. The cardiac silhouette is usually normal on chest x-ray. The underlying hemodynamic deficit is impaired diastolic filling (similar to hypertrophic cardiomyopathy) in contrast to the impaired contractility of dilated cardiomyopathy.

24
Q

A previously healthy 47-year-old woman comes to medical attention because of loss of sensation in her right hand for 2 days. Physical examination is unremarkable other than confirming the presence of right hand hypoesthesia. CT and MRI scans demonstrate a focal lesion in the left parietal cortex. Laboratory investigations show high levels of antiphospholipid antibodies, prolonged activated partial thromboplastin time (aPTT), and normal prothrombin time (PT). Which of the following complications is most likely to develop?

(A) Glomerulonephritis
(B) Lymphoma
(C) Pulmonary fibrosis
(D) Recurrent bleeding
(E) Recurrent thrombosis

A

Respuesta: E

The correct answer is E. Prolongation of aPTT in patients with antiphospholipid antibody syndrome is related to an in vitro reaction of such antibodies with phospholipids present in the reaction test. Surprisingly, these patients are not prone to developing recurrent bleeding (choice D), but instead have hypercoagulability. The resultant propensity to develop thrombosis, both in the arterial and in the venous channels, may lead to fatal cerebral, myocardial, and intestinal infarcts, as well as to recurrent venous thrombosis. This syndrome may be associated with specific collagen vascular diseases (most commonly with systemic lupus erythematosus) or occur as an isolated condition. Other manifestations include renal disease due to microangiopathy, and repeated miscarriage.

Glomerulonephritis (choice A) is not a feature of antiphospholipid antibody syndrome. Systemic lupus erythematosus causes glomerulonephritis of varying forms and severity in up to 90% of patients.

Lymphoma (choice B) is not a complication of antiphospholipid antibody syndrome. An autoimmune-mediated disease leading to increased risk of lymphoma is Sjögren syndrome.

Pulmonary fibrosis (choice C) may develop in association with several immune-mediated collagen vascular diseases, especially rheumatoid arthritis and progressive systemic sclerosis.

25
A 50-year-old woman is admitted for urinary tract infection, complicated by profound weakness, abdominal pain, vomiting, and diarrhea. Her temperature is 40.0 C (104 F), blood pressure is 90/60 mm Hg, pulse is 110/min, and respirations are 18/min. Examination reveals signs of dehydration and skin hyperpigmentation over the elbows and knees. Urinalysis shows pyuria. Laboratory studies show hyponatremia, hyperkalemia, and hypoglycemia. Complete blood count is remarkable for eosinophilia. While waiting for the results of urine and blood cultures, intravenous corticosteroids, broad-spectrum antibiotics, and volume replacement therapy is instituted. In addition to such treatment, which of the following is the most appropriate next step? (A) Cortisol level (B) Cosyntropin stimulation test (C) IV mineralocorticoids (D) Oral corticosteroids (E) Renal ultrasonography
**Respuesta: A** The correct **answer is A.** This patient probably has acute adrenal insufficiency, or adrenal crisis. Measurement of serum cortisol will verify the likely diagnosis (cortisol levels would normally be increased following the stress of surgery). Adrenal insufficiency occurs in patients with latent Addison disease who have stress due to events such as surgery, infections, or severe trauma. Other common situations that may precipitate an adrenal crisis include bilateral adrenalectomy, removal of a cortical adenoma that has suppressed the normal gland, and abrupt cessation of exogenous corticosteroid therapy. Withdrawal of corticosteroid treatment should be carried out by gradually tapering the doses. Acute adrenal insufficiency is characterized by signs and symptoms similar to Addison disease, but manifesting with dramatic severity. Indeed, this is a life-threatening condition requiring emergency administration of IV hydrocortisone (100-300 mg), lest the patient develop irreversible shock. Naturally, fluid infusion should be concomitantly administered, and treatment of the underlying cause (in this case, infection) carried out. The cosyntropin stimulation test (**choice B**) is used to confirm a diagnosis. However, when the clinical picture is so severe, as in this example, hydrocortisone treatment takes precedence over diagnostic confirmation. IV mineralocorticoids (**choice C**) are not necessary when such large doses of hydrocortisone are administered, as glucocorticoids also have mineralocorticoid activity. Oral corticosteroids (**choice D**) would not be appropriate in a medical emergency such as adrenal crisis. They are used once the patient’s condition has stabilized. Further, patients in adrenal crisis most often are unable to take anything by mouth because of nausea and vomiting. Renal ultrasonography (**choice E**) is used to study the kidney and collecting system to screen for such conditions as hydronephrosis and renal masses. It would be hardly justifiable in the present case.
26
A 62-year-old man with a long history of cigarette smoking goes to a physician because of a drooping right eyelid. The patient denies headache or weight loss. He complains of an occasionally productive cough but is otherwise in good health. Examination shows right ptosis and a small pupil. Extraocular movements and visual acuity are normal. The right side of his face appears warm and dry. Which of the following is the most appropriate step in diagnosis? (A) Chest x-ray examination (B) Laboratory testing for syphilis (C) MRI scan of the head (D) Ophthalmologic referral (E) Tonometric measurement
**Respuesta: A** The correct **answer is A**. This patient is exhibiting the signs of Horner syndrome, resulting from damage to the cervical sympathetic plexus that contains presynaptic fibers to the superior cervical ganglion. This results in deficient sympathetic innervation to the iris and the tarsal muscles of the eye, as well as to the sweat glands and vessels in the ipsilateral face. This deficit leads to miosis and ptosis in the ipsilateral eye, and anhidrosis and redness of the ipsilateral hemiface. The most common cause of this syndrome is pulmonary cancer of the apex, which may spread to the sympathetic plexus by contiguity. A patient presenting with Horner syndrome, especially if a smoker, should undergo radiologic investigations to exclude a lung tumor. Laboratory testing for syphilis (**choice B**) would be appropriate in the presence of other neurologic signs of syphilitic involvement of the nervous system, such as impaired proprioception and vibration sense. Argyll-Robertson pupil is a manifestation of tabes dorsalis. The pupils are small and poorly reactive to light but normally reactive to accommodation. There is no indication that any type of intracranial pathology is responsible for the ocular changes seen in this case; thus, MRI scan of the head (**choice C**) would not be a useful diagnostic method. Ophthalmologic referral (**choice D**) would certainly be appropriate for more complex symptomatology, but classic Horner syndrome is an easy diagnosis, which should, without delay, prompt a search for the underlying cause. Tonometric measurement (**choice E**) is appropriate in the presence of signs suggesting narrow-angle (acute) glaucoma, including severe unilateral eye pain, redness, a dilated and fixed pupil, and blurring of vision.
27
A 60-year-old white man is evaluated by a physician because of weight gain and increasing abdominal girth of several months’ duration, despite no change in diet. His temperature is 37.1 C (98.8 F), blood pressure is 70/40 mm Hg, pulse is 120/min, and respirations are 18/min. Head and neck examination is remarkable for “bags” under the eyes and jugular venous distension. Chest examination demonstrates pulmonary rales and a third heart sound. Abdominal examination reveals an enlarged, tense abdomen with a palpable fluid wave; abdominal organ palpation is inadequate because of the tension. Examination of the extremities demonstrates pitting edema at the ankles. Which of the following is the most likely diagnosis? (A) Alcoholic cirrhosis (B) Chronic hepatitis B infection (C) Congestive heart failure (D) Gastric carcinoma (E) Hypothyroidism
**Respuesta: C** The correct **answer is C**. Although cirrhosis usually comes to mind first when you think of ascites, you should remember that ascites can have a variety of causes, both hepatic and nonhepatic. Hepatic causes include cirrhosis (especially alcoholic), chronic hepatitis, severe alcoholic hepatitis without cirrhosis, and BuddChiari syndrome (hepatic vein obstruction). Nonhepatic causes include generalized fluid retention (heart disease, nephrotic syndrome, severe hypoalbuminemia, renal failure), intraabdominal disease (cancer or tuberculosis involving the peritoneum; rarely pancreatitis), and, rarely, hypothyroidism. A useful rule of thumb is that ascites isolated from or disproportionate to peripheral edema is usually due to liver or intra-abdominal disease, whereas systemic diseases such as congestive heart failure tend to produce both peripheral edema and ascites. The clinical findings illustrated in the question stem are typical of congestive heart failure. Alcoholic cirrhosis (**choice A**) is the most common form of cirrhosis, but it is not usually accompanied by severe peripheral edema. Chronic hepatitis B infection (**choice B**), even in the absence of cirrhosis, can cause ascites, but it is not usually accompanied by severe peripheral edema. Intra-abdominal cancers (**choice D**) that have spread to the peritoneum can cause an ascites that is usually not accompanied by severe peripheral edema. Hypothyroidism (**choice E**) is a rare cause of significant ascites that is usually accompanied by other stigmata of hypothyroidism, such as coarse hair and facial puffiness with periorbital edema and eyelid droop.
28
A 65-year-old man comes to the physician because of progressive severe shortness of breath, weight loss, and left chest pain. Chest examination reveals dullness to percussion and diminished breath sounds over the left hemithorax. A chest x-ray film shows irregular nodular thickening of the visceral pleura, with partial obliteration of the pleural sac and modest pleural effusion. The underlying lung parenchyma is minimally involved. Cytologic examination of the pleural effusion is positive for malignant cells. Which of the following is the most likely predisposing factor for this patient’s condition? (A) Cigarette smoking (B) Occupational asbestos exposure (C) Occupational coal exposure (D) Occupational silica exposure (E) Prior radiation therapy
**Respuesta: B** The correct **answer is B**. The clinical picture is consistent with mesothelioma, a cancer of mesothelial origin that manifests as a diffuse plaque-like thickening of the pleura. Peritoneal mesotheliomas are much more uncommon. Most mesotheliomas coming to clinical attention nowadays result from prior occupational exposure to asbestos, which was associated with shipyard work, insulation, brake lining, building and roof work, and mining, among others. Inhalation of asbestos fibers is also a risk factor for other cancers, including bronchogenic, laryngeal, and colon carcinoma. The main differential diagnosis is with bronchogenic carcinoma, which may spread to the pleura but is usually accompanied by one or more intrapulmonary nodules. Cigarette smoking (**choice A**) does not seem to predispose to the development of mesotheliomas. However, it does increase the risk of bronchogenic carcinoma in persons exposed to asbestos. Occupational exposures to coal (**choice C**) and silica (**choice D**) are associated with coal-worker pneumoconiosis and silicosis, respectively, but not mesotheliomas. Prior radiation injury (**choice E**) to the lungs may be followed by two conditions: acute radiation pneumonitis (occurring 2-3 months after radiation) and interstitial fibrosis (manifesting 6-12 months after exposure). Radiation is not a predisposing factor for either bronchogenic carcinoma or mesothelioma.
29
A 60-year-old woman consults a physician because of recurrent nose bleeds. Physical examination of the nose demonstrates no obvious mucosal abnormalities or masses, but the mild trauma of the examination causes a nosebleed that persists for 10 minutes. A complete blood count is performed, showing a platelet count of 600,000/μL. Which of the following additional findings would most strongly support essential thrombocythemia as the cause of this woman’s elevated platelet count? (A) Giant platelets (B) Increased bone marrow fibrosis (C) Increased RBC mass (D) Positive Philadelphia chromosome (E) Tear-drop shaped RBCs
**Respuesta: A** The correct **answer is A**. An increased platelet count is more formally called thrombocythemia. Thrombocythemia can occur in primary form (essential thrombocythemia), as part of other myeloproliferative disorders (e.g., polycythemia vera, chronic myelogenous leukemia, and idiopathic myelofibrosis), or secondary to other processes such as acute infection, chronic inflammatory disorders (rheumatoid arthritis, inflammatory bowel disease, tuberculosis, sarcoidosis, Wegener granulomatosis), hemorrhage, iron deficiency, or tumors (cancers and lymphomas). Essential thrombocythemia is a clonal abnormality of a hemopoietic stem cell that is seen most often in patients between 50 and 70 years of age. The platelet count may reach 1,000,000/μL or higher, but may also be in a more ambiguous range down to about 500,000/μL. Helpful clinical clues on the peripheral smear are platelet aggregates, giant platelets, and megakaryocyte fragments. Increased bone marrow fibrosis (**choice B**) and tear-drop shaped RBCs (**choice E**) are features of idiopathic myelofibrosis, which can also have elevated platelet counts. Increased RBC mass (**choice C**) is a feature of polycythemia vera, which can also show elevated platelet counts. The Philadelphia chromosome (**choice D**) is a feature of chronic myelogenous leukemia, which can also show elevated platelet counts.
30
A 50-year-old alcoholic woman is brought to the emergency department 24 hours following a suicide attempt by ingestion of 5 g acetaminophen and 80 mg diazepam. A patient’s neighbor relates that she had been drinking heavily for the past week. The patient is drowsy and difficult to arouse. Her temperature is 36.5 C (98 F). Physical examination reveals mild jaundice, fetor hepaticus, and gingival bleeding. Laboratory studies show: - AST 2500 U/L - ALT 3000 U/L - Bilirubin, total 2.5 mg/dL - Prothrombin time 23 sec Which of the following is (are) the most important factor(s) leading to this patient’s hepatic disease? (A) Acetaminophen alone (B) Alcohol alone (C) Alcohol and acetaminophen (D) Diazepam alone (E) Diazepam and alcohol
**Respuesta: C** The correct **answer is C**. This patient manifests signs and symptoms of fulminant hepatic failure. Infectious hepatitis due to hepatitis B virus (HBV) is responsible for half the cases of fulminant hepatitis in the U.S. Fulminant hepatic failure carries a 60% mortality rate. Drugs are the most frequent nonviral causes of fulminant hepatic failure. Acetaminophen produces liver injury in a dose-dependent fashion. Acute acetaminophen overdose usually manifests with doses higher than 7 g in adults, but concomitant alcohol abuse lowers the threshold of hepatic toxicity. Patients with chronic alcohol abuse may develop acute diffuse liver necrosis (the pathologic substrate of fulminant hepatic failure) after taking as little as 4 g acetaminophen within a 24-h period. The synergistic effect between alcohol and acetaminophen is mediated by the activating action of alcohol on cytochrome P-450, which converts acetaminophen into a highly toxic intermediate. This intermediate depletes hepatic glutathione stores, resulting in increasing concentration of free radicals and consequent cellular toxicity. Acetaminophen alone (**choice A**) is also capable of producing significant hepatotoxicity, especially in children. The patient’s history suggests, however, that alcohol played a contributing role. Alcohol alone (**choice B**) is also a hepatotoxic drug and in large amounts may cause acute alcoholic hepatitis. However, fulminant hepatic failure exclusively related to alcohol toxicity is exceedingly rare. Diazepam alone (**choice D**) does not explain the clinical and laboratory signs of hepatic failure. Diazepam overdose results in prolonged sedation, which usually resolves with supportive measures. Respiratory depression is the most significant and potentially life-threatening consequence. This is particularly likely when diazepam (or similar benzodiazepine) is ingested in association with alcohol (**choice E**).
31
An otherwise healthy 20-year-old man is referred for genetic counseling because of a family history of multiple endocrine neoplasia (MEN) type IIa. He elects to undergo genetic testing. He is found to have a mutation of the RET proto-oncogene. Which of the following is the most appropriate next step in management to prevent morbidity and mortality? (A) Periodic calcitonin level measurements (B) Periodic screening with imaging studies (C) Periodic screening for latent pheochromocytoma (D) Prophylactic parathyroidectomy (E) Prophylactic total thyroidectomy
**Respuesta: E** The correct **answer is E**. Genetic screening for MEN type II is available to persons with a family history of this condition. Mutations of the RET proto-oncogene make these patients more likely to develop all the endocrine conditions associated with MEN type IIa, including pheochromocytoma, parathyroid hyperplasia/adenoma, and medullary carcinoma of the thyroid. Medullary carcinoma of the thyroid is an aggressive cancer arising from calcitonin producing cells (C cells) in the thyroid. Although penetrance of this autosomal dominant genetic mutation is incomplete, the risk of developing medullary carcinoma warrants prophylactic thyroidectomy. Recall that MEN IIb is similar to MEN IIa, except for the additional presence of mucosal neuromas and a Marfan-like body habitus. If persons with family history of MEN type II decline genetic testing, they may be offered the possibility of screening for early medullary carcinoma by calcitonin measurements (**choice A**) following pentagastrin stimulation. Peaks greater than 190 pg/mL in males or 80 pg/mL in females are consistent with medullary carcinoma. Periodic screening with imaging studies (**choice B**) is not a valuable option to detect early medullary carcinoma, since an early lesion may be too small to be seen on CT or MRI scans. Periodic screening for latent pheochromocytoma (**choice C**) is not cost-effective, since pheochromocytomas develop less frequently than medullary carcinomas and are usually benign. However, screening for latent pheochromocytoma (urinary catecholamine levels) should be carried out before performing any major surgical procedure to avoid a hypertensive crisis. Prophylactic parathyroidectomy (**choice D**) is not indicated. Hyperplasia or adenoma of the parathyroids may lead to hyperparathyroidism, in which case parathyroidectomy may be performed.
32
A 24-year-old woman consults a physician because of difficulty swallowing both solid foods and liquids. She first noted occasional difficulty with swallowing 3 years ago, but states that the problem is now much more severe and persistent and is accompanied by regurgitation. Often the food she regurgitates 1 to 2 hours after a meal still tastes like “normal food.” She has lost approximately 10 pounds, which she relates to uncomfortable feelings in her chest when she eats. Barium x-ray demonstrates a strikingly dilated esophagus with marked narrowing at the lower esophageal sphincter. Esophageal manometry shows aperistalsis. The lower esophageal sphincter pressure is increased, and there is often incomplete relaxation on swallowing. Which of the following is the most likely diagnosis? (A) Achalasia (B) Esophageal cancer (C) Esophageal web (D) Lower esophageal ring (E) Symptomatic diffuse esophageal spasm
**Respuesta: A** The correct **answer is A**. This is a classic presentation of achalasia, which is an esophageal disorder characterized by impaired esophageal peristalsis and impaired relaxation of the lower esophageal sphincter. It is thought to be due to malfunction of the esophageal myenteric plexus. Achalasia can occur at any age, but most frequently presents in the 3rd or 4th decade. Achalasia can be complicated by (usually nocturnal) pulmonary aspiration of regurgitated food, life-threatening esophageal rupture, and a questionably increased incidence of esophageal cancer. Treatment of achalasia is by forceful dilation of the lower esophageal sphincter. Sublingual nitroglycerin before meals and calcium channel blockers are sometimes used to prolong the times between lower esophageal forced dilation. More aggressive therapies, such as injection of botulinum toxin in the lower esophageal sphincter or myotomy of the lower esophageal sphincter, are reserved for patients who do not respond to other therapies. Esophageal cancer (**choice B**) would produce an area of ulceration or a mass in the esophagus, which would have been picked up on the barium swallow. An esophageal web (**choice C**) is a thin mucosal web that grows across the lumen of the esophagus, typically in the upper esophagus. It would have been detected on the barium studies. A lower esophageal (Schatzki) ring (**choice D**) is a 2 to 4-mm mucosal ring near the squamocolumnar junction that is probably of congenital origin and usually causes only relatively mild difficulty with swallowing of solids. Symptomatic diffuse esophageal spasm (**choice E**) differs from achalasia in that there are strong, but poorly coordinated and ineffective, esophageal contractions, rather than aperistalsis. Over many years, it may evolve into achalasia, but the disorders are clinically considered distinct.
33
A patient consults a physician because of chronic nasal congestion and nosebleeds over the past 2 years. He has tried numerous overthe-counter medications, but nothing has helped. He has also felt chronically ill, with lowgrade fever, malaise, and anorexia. Nasal examination demonstrates a friable mucosa with a red, raised granular appearance that bleeds easily. A small area of nasal septal perforation is seen. Chest x-ray film demonstrates pulmonary infiltrates with associated cavitation in one case. Which of the following is the most appropriate next step in diagnosis? (A) Abdominal CT scan (B) Liver enzymes (C) Pancreatic enzymes (D) Urinalysis (E) Bladder biopsy
**Respuesta: D** The correct **answer is D.** The patient has both significant nasal disease and significant pulmonary disease. This should suggest the possibility of Wegener granulomatosis (WG), which is a granulomatous inflammation of the upper and/or lower respiratory tract that can also destroy the kidneys. Nasal or pulmonary biopsy confirms the clinically suspected diagnosis by showing granulomas and a perivascular inflammatory infiltrate. If renal damage has occurred, urinalysis typically shows proteinuria, hematuria, and red cell casts. The clinical presentation illustrated is a common variant. Other patients may present with systemic symptoms mimicking cancer (malaise, fever, weight loss, anorexia), migratory polyarthritis, granulomatous skin lesions, or ocular manifestations. WG formerly had a dismal prognosis, but now can be well controlled with immunosuppressive cytotoxic drugs. Abdominal CT scan (**choice A**) would probably not show anything of significance. Liver (**choice C**) or pancreatic (**choice D**) enzymes would not be expected to be altered in this condition. Bladder biopsy (**choice E**) would probably not show anything of significance; however, a renal biopsy might be very helpful.
34
A 55-year-old woman with hypertension presents to her primary care physician with 3 weeks of worsening headaches and language difficulty. She reports severe headaches that waken her from sleep and are worse in the morning, frequently associated with nausea. She complains of slowness of speech and difficulty with wordfinding. She is afebrile. A neurologic examination is remarkable for slow, effortful speech with poor repetition, and a right pronator drift. The physician refers her to the emergency department for an urgent MRI study, which shows a diffusely enhancing intra-axial abnormality in the left frontal region. Which of the following is the most likely diagnosis? (A) Astrocytoma (B) Brain abscess (C) Cerebral infarct (D) Complicated migraine (E) Meningioma
**Respuesta: A** The correct **answer is A**. Astrocytoma typically presents in the 40- to 60-year age range with headaches, focal neurologic symptoms, and seizures. Headaches from tumors may waken patients from sleep and are worse in the morning, presumably from increased intracranial pressure after being supine over night. Highgrade astrocytomas tend to enhance diffusely on imaging. Brain abscesses (**choice B**) may present with headache and focal findings but typically progress more rapidly. Fever is a prominent component. Imaging shows a ring-enhancing lesion. Cerebral infarcts (**choice C**) rarely cause headaches. The clinical course would not be progressive but would begin acutely and then stabilize. Infarcts may show diffuse enhancement after several weeks. Complicated migraine (**choice D**) refers to migraine headache with a focal but transient neurologic deficit, which may include language difficulty or hemiparesis. Migraine headaches do not typically waken people from sleep. Patients with complicated migraine have an increased risk of stroke, but imaging studies during a complicated migraine are usually normal. Meningiomas (**choice E**) are more common in women and may present with headaches and focal features from mass effect. The course tends to be more chronic, however, and imaging will demonstrate an extra-axial structure compressing the underlying brain.
35
A 22-year-old woman presents with an 8-month history of bilateral and symmetric polyarthritis affecting the proximal small joints of the hands, wrists, elbows, knees, hips, and ankles. Morning stiffness is prominent. Marginal bony erosions are noted on radiologic examination. The erythrocyte sedimentation rate (ESR) is 66 mm/hr, and rheumatoid factor is positive. She has been treated with paracetamol and ibuprofen (600 mg, 2-3 times a day) with minimal relief. Clinical examination confirms the presence of fluid and swelling of the joints. Which of the following is the most appropriate management strategy? (A) Anti-inflammatory doses of nonsteroidal anti-inflammatory drugs (NSAIDs) (B) NSAIDs and a disease-modifying antirheumatic drug (DMARD) (C) NSAIDs with low-dose narcotic analgesics (D) NSAIDs and low-dose prednisone
**Respuesta: B** The correct **answer is B**. This patient seems to have aggressive erosive disease and requires a disease-modifying (slow- acting antirheumatic) drug (DMARD). There are numerous choices, including methotrexate, cyclosporine, minocycline, and anticytokine (tumor necrosis factor) therapies. Anti-inflammatory doses of NSAIDs are also generally required. Short-term use of prednisone may be of use to control inflammation rapidly, but only in conjunction with disease-modifying therapy. Longer-term, low-dose (less than 10 mg) use is common in as many as one fourth of patients seen in hospital-based clinics because of inadequately controlled inflammation. Anti-inflammatory therapy alone can allow progression of erosive joint disease. NSAIDs alone (**choice A**) are inappropriate in this case because of the reasons noted above. Pain relief should occur through control of inflammation, rather than the use of narcotics (**choice C**). The efficacy of narcotics alone in inflammatory arthritic pain is limited. NSAIDs with prednisone (**choice D**) may be very effective for controlling overt symptoms but not for preventing disease progression
36
A 55-year-old alcoholic man is brought to the emergency department by the police after being found wandering and mumbling to himself. The man is unable to give a coherent history. The initial impression is of an emaciated, jaundiced, and confused man who appears older than his stated age. Vital signs are stable and within normal limits. The breath has a musty, sweet odor. Abdominal examination shows ascites and marked nodularity of the liver edge. A “caput medusa” is seen near the umbilicus. Neurologic examination is notable for asterixis. A toxicology screen is negative. Aspartate aminotransferase (AST), alanine aminotransferase (ALT), and blood ammonia are all moderately increased. The man is admitted to the hospital and given an extremely low protein diet with oral carbohydrate supplementation. The bowels are cleared with an enema. Which of the following is the most appropriate pharmacotherapy? (A) Ampicillin, oral (B) Benzathine penicillin, intramuscular (C) Ceftriaxone, oral (D) Neomycin, oral (E) Penicillin G, IV
**Respuesta: D** The correct **answer is D**. This patient has hepatic encephalopathy (hepatic coma, portal-systemic encephalopathy), a neuropsychiatric syndrome caused by liver disease. It is usually associated with portal-systemic shunting of venous blood, which can cause esophageal varices and dilation of veins near the umbilicus (“caput medusa”). The diagnosis is usually made on clinical, rather than laboratory, grounds, liver function tests correlate poorly. Serum ammonia level is usually elevated, but specific values correlate poorly with clinical status. Therapy is based on removing sources of nitrogen (e.g., protein) in the gut by enema, restriction of dietary protein, and reduction of bacterial load (since some bacteria produce ammonia). Two different strategies are often used for reducing the bacterial load: oral lactulose, which acts as an osmotic cathartic to “wash the bacteria out,” and oral neomycin (a poorly absorbed aminoglycoside), which can be used to kill most of the bacteria while minimizing significant systemic side effects. (Oral neomycin is also sometimes used as part of bowel preparation prior to abdominal surgery.) Other types and routes of administration of antibiotics are not usually used in this setting. In some hospitals, oral neomycin is initially used for bacterial load reduction. Patients are then switched to longer-term lactulose, thereby limiting the potential nephrotoxicity and ototoxicity of the neomycin. Oral ampicillin (**choice A**) is a commonly used antibiotic in outpatient settings. Benzathine penicillin (**choice B**) is an intramuscular, depot form of penicillin that is used most often for syphilis and for month-long prophylaxis against recurrent rheumatic fever. Oral ceftriaxone (**choice C**) is a commonly used third-generation cephalosporin. IV penicillin G (**choice E**) is reserved for treatment of patients with sensitive organisms in a hospital setting.
37
A 69-year-old man with history of congestive heart failure secondary to hypertension and atrial fibrillation comes to the physician because of increasing nausea, headache, and blurred vision in both eyes over the past 3 days. He also complains of seeing “yellow halos around lights.” His medications include hydrochlorothiazide, enalapril, and digoxin. His blood pressure is 132/84 mm Hg, pulse is 56/min with irregular rhythm, and respirations are 16/min. Which of the following is the most likely cause of these symptoms? (A) Acute narrow-angle glaucoma (B) Digoxin toxicity (C) Diuretic-induced hypokalemia (D) Enalapril overdose
**Respuesta: B** The correct **answer is B**. The symptoms (particularly xanthopsia/verdopsia = a yellow-green cast to the vision) are characteristic of digitalis toxicity. Bradycardia and different types of arrhythmias are also effects of excess digitalis. The medication should be immediately discontinued in these cases. Acute narrow-angle glaucoma (**choice A**) manifests with unilateral severe eye pain and redness, accompanied by blurred vision and halos around lights. Diuretic-induced hypokalemia (**choice C**) is a potential risk with thiazide and loop diuretics. Intravascular volume depletion, with resultant prerenal azotemia, hyperglycemia, hyperuricemia, and hepatic dysfunction, can also occur. Enalapril overdose (**choice D**) would manifest with hypotension, which is not present in this case. ACE inhibitors have been shown to improve survival and now represent a mainstay in the treatment of congestive heart failure. They decrease ventricular preload by causing venous dilatation and diminishing venous return.
38
A 45-year-old white woman consults a physician because of chronic weakness, fatigue, and dizziness on standing. The patient frequently feels nauseous and sometimes vomits and has diarrhea. Physical examination demonstrates diffuse tanning of the skin, even in areas not exposed to sun, which is most pronounced over bony prominences. Scattered black freckles are seen on the head and neck, as are occasional patches of vitiligo. The areola and the mucous membranes of the mouth and vagina have bluish-black discoloration. Which of the following is the most likely diagnosis? (A) Addison disease (B) Conn disease (C) Cushing disease (D) Graves disease (E) Hashimoto disease
**Respuesta: A** The correct **answer is A**. This is an example of Addison disease, which can occur in all ages and both sexes and is caused by the destruction of the adrenal cortexes due to processes such as autoimmune disease, tuberculosis, infarction, and cancer. Although the symptoms of Addison disease (chronic adrenocortical insufficiency) are very nonspecific and can easily resemble other endocrine disorders (notably hypothyroidism), significant hyperpigmentation can be a helpful diagnostic clue. With the exception of adrenal insufficiency due to pituitary failure, pigmentation is usually increased because of large amounts of ACTH, which has some activity in stimulating melanocytes. Other symptoms and signs of Addison disease that are not mentioned in the question stem include a decreased tolerance to cold, ECG changes (decreased voltage and prolonged PR), EEG changes (generalized slowing of alpha rhythm), and, in later stages, weight loss, dehydration, hypotension, and small heart size. Adrenal crises, characterized by severe abdominal pain and cardiovascular collapse, may complicate septicemia, trauma, and operative procedures. Conn disease (**choice B**) is primary aldosteronism. It is characterized by episodic weakness, paresthesias, transient paralysis, tetany, and diastolic hypertension. Cushing disease (**choice C**) is hypercortisolism due to a pituitary adenoma. It is characterized by rounded facies, buffalo hump, muscle wasting, thin skin, abdominal striae, hypertension, glucose intolerance, and reduced resistance to infection. Graves disease (**choice D**) is an autoimmune cause of hyperthyroidism, with heat intolerance, palpitations, tachycardia, tremor, and ophthalmopathy. Hashimoto disease (**choice E**) is an autoimmune cause of (usually) hypothyroidism, with cold intolerance, rubbery goiter, facial puffiness, and sparse, coarse hair.
39
A 26-year-old woman presents to her physician after noticing a neck mass. Physical examination reveals a large thyroid gland that is firm, multilobular, and mobile. Her serum thyroid-stimulating hormone (TSH) is elevated, but her serum triiodothyronine (T3) and thyroxine (T4) are low. Her serum antithyroglobulin titer is positive. A thyroid scan reveals non-uniform uptake. A biopsy of the thyroid mass is most likely to show which of the following? (A) Fibrosis (B) Granulomas (C) Lymphocytic infiltration (D) Neutrophilic infiltration (E) Parafollicular (“C”) cell hyperplasia
**Respuesta: C** The correct **answer is C**. This patient most likely has Hashimoto thyroiditis, which is the most common cause of thyroiditis in the U.S., with a predominance in women. Patients notice a goiter but are initially euthyroid. Needle biopsy will reveal a lymphocytic infiltration, and serum analysis typically shows antibodies directed against thyroglobulin. Fibrosis (**choice A**) is seen in Riedel struma, also known as Riedel fibrosing thyroiditis. This disorder is associated with intense fibrosis of the thyroid, leading to induration of neck tissues. The principal importance of this disorder is that it requires differentiation from thyroid neoplasia. Granulomas (**choice B**) with multinucleate giant cells are seen in subacute granulomatous (de Quervain) thyroiditis. Neutrophils (**choice D**) would be seen in subacute granulomatous (de Quervain) thyroiditis, which usually follows an upper respiratory infection. Symptoms include malaise and pain over the thyroid gland referred to the lower jaw. Patients have a high erythrocyte sediment rate (ESR). Serum T3 and T4 are high and, TSH is undetectable. Parafollicular hyperplasia (**choice E**) is associated with medullary thyroid cancer. Primary thyroid carcinomas may be classified into two varieties, depending on whether the lesions arise from the thyroid follicular epithelium or from the parafollicular, or C, cells. The mainstay of therapy is surgical excision; external radiation and chemotherapy have a palliative role for excision for recurrent or residual disease.
40
A 43-year-old woman who has recently moved to the United States from Greece goes to the local clinic for her annual physical examination. She suffers from anemia which was diagnosed during her first pregnancy at the age of 19, for which she has taken an iron pill on and off throughout her entire adult life. She says her hemoglobin levels have never been less than 10 g/dL and she has never needed any blood transfusion. She reports that she has not taken her iron pills in over a year and she feels well. She had a hysterectomy about 3 years ago but her anemia continued the same even after her menses stopped. On examination, the patient’s vital signs are all within normal limits and the rest of her examination is unremarkable. A Pap smear and routine mammogram are ordered. Laboratory studies show: - Hemoglobin 10.8 g/dL - Platelet count 250,000/mm3 - MCH 28 g/dL - MCV 69 fL - RDW 13.9% - Leukocytes 5600/mm3 Two weeks later the patient comes in for a follow-up visit and to review the results of her lab tests. Which of the following management options is of the greatest diagnostic value at this time? (A) Order a hemoglobin electrophoresis (B) Order a bone marrow biopsy (C) Prescribe oral ferrous sulfate (D) Order vitamin B12 and folate levels (E) Order iron, ferritin, and total iron binding capacity
**Respuesta: A** The correct **answer is A**. This patient clearly has an anemia that is long-standing and is not related to her menses, considering that she had a hysterectomy and the anemia persists, plus this patient is of Mediterranean descent. The most likely cause of her anemia is an abnormal hemoglobin, such as thalassemia, because her RDW is less than 15%. Therefore, the most appropriate management option for this patient would be a hemoglobin electrophoresis. Bone marrow biopsy (**choice B**) in patients with microcytic anemia is indicated only if Sideroblastic anemia were suspected; in this patient, the diagnosis of sideroblastic anemia is not considered. Prescribing oral ferrous sulfate (**choice C**) or ordering iron, ferritin, and total iron binding capacity (**choice E**) would be incorrect. If this patient had iron deficiency anemia her RDW would be above 15%. If iron deficiency anemia were suspected, then iron studies would be of diagnostic value. Ordering B12 and folate levels (**choice D**) is not indicated because this is a microcytic anemia; B12 and folate deficiency would produce a macrocytic anemia.
41
A 40-year-old man with type 1 diabetes mellitus for 10 years was recently found to have mild hypertension (stage 1) and microalbuminuria. Renal function tests are otherwise normal. There is evidence of mild left ventricular hypertrophy on ECG and echocardiographic studies. Blood lipids are within normal limits. Which of the following is the most appropriate pharmacologic treatment for this case of hypertension? (A) ACE inhibitors (B) Beta blockers (C) Calcium channel blockers (D) Central alpha-agonist, e.g., clonidine (E) Thiazide diuretics
**Respuesta: A** The correct **answer is A**. The most appropriate pharmacologic treatment of hypertension in diabetic patients is an ACE inhibitor, especially if there is concomitant evidence of diabetic glomerulopathy (i.e., microalbuminuria). Studies have shown that ACE inhibitors exert a beneficial effect on diabetic nephropathy by lowering the intraglomerular capillary pressure. Thus, ACE inhibitors are also used to slow the progression of asymptomatic diabetic glomerulonephropathy, even in the absence of systemic hypertension. Most studies have suggested that monotherapy with either a beta blocker (**choice B**) or a thiazide diuretic (**choice E**) is the most appropriate initial treatment for uncomplicated hypertension, i.e., hypertension without detectable end-organ damage. Calcium channel blockers (**choice C**) are frequently used as firstline antihypertensive agents. These drugs are also beneficial in case the patient has asthma, migraine, or urinary incontinence. However, recent studies have suggested an association between use of calcium channel blockers and cardiac complications. Central alpha-agonists, e.g., clonidine (**choice D**), are seldom used as first-line agents for treatment of hypertension because of their undesirable side effects. Methyldopa is frequently used in pregnancy.
42
A 28-year-old African American woman comes to the emergency department complaining of chest and abdominal pain that is rated an 8 out of 10. The pain is severe enough to cause her to vomit repeatedly. She tells you that she has had episodes such as this in the past that are associated with her sickle cell disease, and that she thinks this pain is similar. For the last year, however, she has been fairly healthy, aside from a respiratory infection that she just got over a few days ago. Her vital signs are: temperature 37.8 C (100 F), blood pressure 158/90 mm Hg, pulse 110/min, and respirations 28/min. Examination reveals a woman in pain but without any specific findings. A complete blood count with a manual slide review reveals a hematocrit of 29%, a white blood cell count of 12,000 cells/mm3 , and platelets of 400,000/mm3 . Numerous crescent-shaped red blood cells, red blood cells with eccentrically located ovoid granules, red blood cells with a dark center surrounded by a light band that again is encircled by a darker ring, and nucleated red blood cells are present. An electrocardiogram reveals sinus tachycardia, and a set of cardiac enzymes drawn in the emergency department is normal. This patient is most likely to benefit from which of the following therapies? (A) Acetaminophen (B) Aspirin (C) Broad-spectrum antibiotics (D) Narcotics and hydration (E) Plasma exchange
**Respuesta: D** The correct **answer is D**. This patient has a sickle cell pain crisis, a condition caused by microvascular occlusive events. The pain is severe and often requires admission for pain control with narcotic level analgesics and rehydration. Acetaminophen (**choice A**) and aspirin (**choice B**) can be adjunct pain medications but are not likely to provide adequate control for a sickle cell pain crisis. Broad-spectrum antibiotics (**choice C**) are inappropriate. Mild infections often can cause a sickle cell crisis. Low-grade fevers, anemia, and leukocytosis are common in a sickle cell crisis and do not indicate sepsis. Plasma exchange (**choice E**) may be used if a sickle cell crisis progresses to multiorgan failure. It is not indicated for an acute pain episode.
43
A previously healthy 25-year-old man comes to medical attention because of generalized edema and a sensation of abdominal fullness. Family and personal medical history are unremarkable. Examination reveals diffuse edema and signs of ascites. Laboratory investigations show: Blood, serum: - Albumin 1.7 g/dL - Triglycerides 260 mg/dL - Cholesterol, total 280 mg/dL - Erythrocyte sedimentation rate 50/min - Glucose 110 mg/dL - BUN 15 mg/dL - Creatinine 1.0 mg/dL Urine: - Protein (24-hour collection) 5 g - Sediment Occasional nonspecific casts - Leukocytes None - Glucose None Which of the following is a frequent complication in this setting? (A) Acute pyelonephritis (B) Acute renal failure (C) Hemorrhages (D) Hypertension (E) Venous thrombosis
**Respuesta: E** The correct **answer is E**. What is illustrated here is a classic example of nephrotic syndrome: abundant urine loss of proteins, which constitutes the first pathophysiologic event in the chain leading to hypoproteinemia; edema due to reduced oncotic pressure; and hyperlipidemia secondary to a compensatory increase in hepatic lipoprotein synthesis. Alterations in plasma proteins often result in an elevated erythrocyte sedimentation rate (ESR). Loss of plasma globulins increases susceptibility to infection. Finally, loss of antithrombin III, protein C, and protein S (circulating anticoagulants) results in predisposition for venous thrombosis. Renal venous thrombosis is particularly dangerous in this setting. Acute pyelonephritis (**choice A**) is not a complication of nephrotic syndrome or any of the diseases associated with it. Infections in the setting of nephrotic syndrome usually involve the respiratory tract and are sustained by gram-positive organisms. Acute renal failure (**choice B**) does not constitute an immediate threat for patients with nephrotic syndrome. Depending on the underlying glomerular disease, progression to chronic renal failure occurs more or less frequently. Hemorrhages (**choice C**) do not frequently occur. Nephrotic syndrome results in abnormal hypercoagulability. Hypertension (**choice D**) is a manifestation of nephritic syndrome, along with hematuria and mild proteinuria (<3 g/day).
44
A 19-year-old woman is evaluated because of a 6-week history of intermittent low-grade fever, weight loss, and night sweats. On physical examination she is found to have enlarged lymph nodes on both jugular chains and supraclavicular areas, and chest CT scan shows a large mediastinal mass. One of the most accessible cervical nodes is excised surgically for biopsy; the pathologist reports the presence of Reed-Sternberg cells. Abdominal CT scan is nondiagnostic and bone marrow biopsy is positive. Which of the following is the most appropriate next step in management? (A) Bilateral neck dissections and surgical mediastinal exploration (B) Bone marrow transplant (C) Radiation therapy to the affected areas (D) Staging laparotomy (E) Systemic chemotherapy
**Respuesta: E** The correct **answer is E**. We already know that this woman has Hodgkin lymphoma with disease in her cervical nodes, mediastinum, and bone marrow (stage IV). The standard initial treatment for her is chemotherapy. Surgical excision of affected nodes (**choice A**) is not indicated for Hodgkin lymphoma. The role of the surgeon in this disease is limited to diagnosis. In visceral lymphoma, on the other hand, surgical resection may be part of the overall therapeutic plan. Bone marrow transplant (**choice B**) may be used in selected patients who have failed standard therapy, but it would not be first-line treatment. Radiation therapy (**choice C**) often is used for localized disease. This woman has systemic, advanced disease that is best treated with chemotherapy. In selected cases both modalities may be combined. Staging laparotomy (**choice D**) is a vanishing tool, rarely used nowadays. When radiation therapy was the main modality and CT scans were not readily available, it was important to know if abdominal disease was present to select the best treatment. Although the abdominal CT scan in this case was nondiagnostic, we already have enough information from the chest CT and the bone marrow biopsy to choose chemotherapy. Abdominal exploration is not needed.
45
Six hours after starting treatment with sulfonamides for acute otitis, a 25-year-old man from southern Italy presents to the emergency department because of acute onset of headache, malaise, and cola-colored urine. He says that he had two similar episodes in Italy: one after eating fava beans, another following treatment with acetylsalicylic acid. Blood and serum studies are significant for: - Red blood cells 1.5 million/mm3 - Hemoglobin 7.5 g/dL - Reticulocytes 6% - Indirect bilirubin 5.2 mg/dL (A) Anemia of chronic disease (B) Aplastic anemia (C) Chronic lymphocytic leukemia (D) Glucose-6-phosphate dehydrogenase deficiency (E) Hairy cell leukemia (F) Hereditary spherocytosis (G) Iron-deficiency anemia (H) Megaloblastic anemia (I) Microangiopathic anemia (J) Myelodysplastic syndrome (K) Myelofibrosis (L) Sickle cell anemia (M) Thalassemia
**Respuesta: D** The correct **answer is D**. This presentation and past medical history are suggestive of hemolytic anemia due to glucose-6- phosphate dehydrogenase (G6PD) deficiency. This X-linked genetic defect makes erythrocytes vulnerable to oxidative damage, which may be triggered by infections, drugs, or naturally occurring substances. About 1 million people are affected by this disorder throughout the world, with the highest prevalence in some Mediterranean regions (Sardinia, Sicily, Greece, and Turkey) and western Africa. In the U.S., G6PD deficiency affects 10 to 15% of African Americans. The clinical symptomatology is characterized by recurring episodes of acute hemolysis. A more severe Mediterranean variant results in chronic hemolysis with prominent reticulocytosis, as well as classic hemolytic crises. Dark-colored urine is due to severe hemoglobinuria. Severe hemolytic episodes may be life-threatening and require urgent blood transfusion. Anemia of chronic disease (**choice A**) complicates chronic disorders, such as rheumatoid arthritis, collagen vascular diseases, cancer, and liver disease. It is a normocytic and normochromic anemia, usually of mild-to-moderate degree. It results from iron sequestration in the bone marrow. Aplastic anemia (**choice B**) is due to bone marrow suppression. Usually, it is seen in conjunction with neutropenia and thrombocytopenia (pancytopenia). Most cases are idiopathic, but drugs are among the most common identifiable causes. Chronic lymphocytic leukemia (CLL; **choice C**) is an indolent form of leukemia that affects middle-aged and elderly individuals. It is characterized by marked lymphocytosis in the blood and bone marrow, resulting from progressive accumulation of monoclonal (but well-differentiated) B lymphocytes. Anemia and thrombocytopenia eventually manifest as a result of bone marrow replacement. Hairy cell leukemia (**choice E**) is a rare form of leukemia due to neoplastic transformation of a subset of B lymphocytes. The designation derives from the characteristic hair-like cytoplasmic projections of leukemic cells. Pancytopenia and massive splenomegaly are consistently present. Hereditary spherocytosis (**choice F**) is characterized by spherocytes (small round erythrocytes lacking central pallor) and reticulocytosis on peripheral blood smears. Such abnormal spherocytes are prone to lysis. Chronic hemolysis ensues, with secondary splenomegaly. The underlying abnormality is a defect in spectrin, a cytoskeletal protein forming the scaffolding of the erythrocytic membrane. Iron-deficiency anemia (**choice G**) is characterized by small, hypochromic erythrocytes (microcytic and hypochromic anemia) and usually results from chronic blood loss. Megaloblastic anemia (**choice H**) is so designated because of the presence of megaloblasts in the bone marrow. Megaloblasts are abnormally large red blood cell precursors. Their counterparts in peripheral blood are macrocytes, large red blood cells having a mean corpuscular volume of up to 130 fL. Macrocytic (megaloblastic) anemia is due to vitamin B12 or folate deficiency. Microangiopathic anemia (**choice I**) is a form of hemolytic anemia due to “mechanical” causes, usually resulting from the presence of widespread small thrombi in the microcirculation or prosthetic cardiac devices. Fragmentation of red blood cells is the crucial pathophysiologic event, which leads to the appearance of characteristic schistocytes in peripheral blood smears. DIC and thrombotic thrombocytopenic purpura are examples of diseases leading to this form of anemia. Myelodysplastic syndrome (**choice J**) is a heterogeneous set of disorders characterized by abnormal maturation of bone marrow stem cells. Cytopenias (affecting at least two blood cell lines) associated with hypercellular bone marrow are characteristic. Myelodysplastic syndromes affect elderly persons, who may come to medical attention because of chronic fatigue or bleeding diatheses. Some subsets of myelodysplastic syndromes have a propensity to leukemic transformation. Myelofibrosis (**choice K**) is characterized by progressive bone marrow fibrosis and resultant pancytopenia. As the spleen takes over hematopoietic functions, massive splenomegaly develops. Teardrop-like erythrocytes are the most peculiar morphologic abnormalities on peripheral blood smears. Thalassemia (**choice M**) is a group of hereditary disorders due to deficient production of one of the globin chains (either alpha or beta, or both). Microcytosis is prominent. Positive family history is usually present. Alpha-thalassemia is seen most frequently in people from southeast Asia, whereas beta-thalassemia is seen most commonly in Mediterranean people. Both forms are found in African Americans. The clinical manifestations depend on whether the individual is heterozygous or homozygous. In the most severe forms, manifestations appear after 6 months of age, with growth failure, jaundice, hepatosplenomegaly, and bone and facial deformities. In carriers of thalassemia trait, microcytosis is often the only sign.
46
A 10-year-old African American boy has had jaundice, splenomegaly, and chronic ulcers over his lower legs since his first year of life. Recurrent episodes of bone pain manifest periodically, especially in concomitance with infectious illnesses. (A) Anemia of chronic disease (B) Aplastic anemia (C) Chronic lymphocytic leukemia (D) Glucose-6-phosphate dehydrogenase deficiency (E) Hairy cell leukemia (F) Hereditary spherocytosis (G) Iron-deficiency anemia (H) Megaloblastic anemia (I) Microangiopathic anemia (J) Myelodysplastic syndrome (K) Myelofibrosis (L) Sickle cell anemia (M) Thalassemia
**Respuesta: L** The correct **answer is L**. Sickle cell anemia results from an autosomal recessive mutation of the beta-globin chain of hemoglobin, resulting in an abnormal form of hemoglobin (HbS). Chronic hemolytic anemia, painful episodes due to vaso-occlusive crises, splenomegaly, chronic ulcers of the legs, and susceptibility to infection by the pneumococci and other bacteria are among its most common manifestations. Hematocrit lower than 30%, sickle-shaped erythrocytes, reticulocytosis, and Howell-Jolly bodies are characteristic hematologic features. Approximately 8% of African Americans are carriers of the HbS gene. The diagnosis rests on electrophoretic demonstration of HbS in peripheral blood. Anemia of chronic disease (**choice A**) complicates chronic disorders, such as rheumatoid arthritis, collagen vascular diseases, cancer, and liver disease. It is a normocytic and normochromic anemia, usually of mild-to-moderate degree. It results from iron sequestration in the bone marrow. Aplastic anemia (**choice B**) is due to bone marrow suppression. Usually, it is seen in conjunction with neutropenia and thrombocytopenia (pancytopenia). Most cases are idiopathic, but drugs are among the most common identifiable causes. Chronic lymphocytic leukemia (CLL; **choice C**) is an indolent form of leukemia that affects middle-aged and elderly individuals. It is characterized by marked lymphocytosis in the blood and bone marrow, resulting from progressive accumulation of monoclonal (but well-differentiated) B lymphocytes. Anemia and thrombocytopenia eventually manifest as a result of bone marrow replacement. Hairy cell leukemia (**choice E**) is a rare form of leukemia due to neoplastic transformation of a subset of B lymphocytes. The designation derives from the characteristic hair-like cytoplasmic projections of leukemic cells. Pancytopenia and massive splenomegaly are consistently present. Hereditary spherocytosis (**choice F**) is characterized by spherocytes (small round erythrocytes lacking central pallor) and reticulocytosis on peripheral blood smears. Such abnormal spherocytes are prone to lysis. Chronic hemolysis ensues, with secondary splenomegaly. The underlying abnormality is a defect in spectrin, a cytoskeletal protein forming the scaffolding of the erythrocytic membrane. Iron-deficiency anemia (**choice G**) is characterized by small, hypochromic erythrocytes (microcytic and hypochromic anemia) and usually results from chronic blood loss. Megaloblastic anemia (**choice H**) is so designated because of the presence of megaloblasts in the bone marrow. Megaloblasts are abnormally large red blood cell precursors. Their counterparts in peripheral blood are macrocytes, large red blood cells having a mean corpuscular volume of up to 130 fL. Macrocytic (megaloblastic) anemia is due to vitamin B12 or folate deficiency. Microangiopathic anemia (**choice I**) is a form of hemolytic anemia due to “mechanical” causes, usually resulting from the presence of widespread small thrombi in the microcirculation or prosthetic cardiac devices. Fragmentation of red blood cells is the crucial pathophysiologic event, which leads to the appearance of characteristic schistocytes in peripheral blood smears. DIC and thrombotic thrombocytopenic purpura are examples of diseases leading to this form of anemia. Myelodysplastic syndrome (**choice J**) is a heterogeneous set of disorders characterized by abnormal maturation of bone marrow stem cells. Cytopenias (affecting at least two blood cell lines) associated with hypercellular bone marrow are characteristic. Myelodysplastic syndromes affect elderly persons, who may come to medical attention because of chronic fatigue or bleeding diatheses. Some subsets of myelodysplastic syndromes have a propensity to leukemic transformation. Myelofibrosis (**choice K**) is characterized by progressive bone marrow fibrosis and resultant pancytopenia. As the spleen takes over hematopoietic functions, massive splenomegaly develops. Teardrop-like erythrocytes are the most peculiar morphologic abnormalities on peripheral blood smears. Thalassemia (**choice M**) is a group of hereditary disorders due to deficient production of one of the globin chains (either alpha or beta, or both). Microcytosis is prominent. Positive family history is usually present. Alpha-thalassemia is seen most frequently in people from southeast Asia, whereas beta-thalassemia is seen most commonly in Mediterranean people. Both forms are found in African Americans. The clinical manifestations depend on whether the individual is heterozygous or homozygous. In the most severe forms, manifestations appear after 6 months of age, with growth failure, jaundice, hepatosplenomegaly, and bone and facial deformities. In carriers of thalassemia trait, microcytosis is often the only sign.
47
A 65-year-old man with a history of cholelithiasis presents with jaundice for 1 week, a 5-kg weight loss for 1 month, and deep epigastric pain for 2 months. He also reports dark urine and claycolored stools. A palpable distended gallbladder is noted on physical examination. Serum chemistry tests show: - Bilirubin Total 5.0 mg/dL - Direct 3.5 mg/dL - Alkaline phosphatase 800 U/L - ALT 45 U/L - AST 40 U/L - Amylase 150 U/L - Glucose (fasting) 150 mg/dL (A) Acute cholangitis (B) Acute cholecystitis (C) Acute pancreatitis (D) Adenoma of the liver (E) Alcoholic hepatitis (F) Amebic liver abscess (G) Focal nodular hyperplasia (H) Gilbert syndrome (I) Hemochromatosis (J) Hepatocellular carcinoma (K) Pancreatic carcinoma (L) Primary biliary cirrhosis (M) Primary sclerosing cholangitis (N) Rotor syndrome (O) Viral hepatitis (P) Wilson disease
**Respuesta: K** The correct **answer is K**. The type of jaundice manifested by this patient is obstructive, as evidenced by mostly conjugated hyperbilirubinemia, high serum levels of alkaline phosphatase, and clay-colored stools. Concomitant weight loss and deep epigastric pain are highly suggestive of pancreatic malignancy. The presence of a distended gallbladder associated with jaundice, known as Courvoisier sign, is also suggestive of cancer. Infiltration of the pancreas may also lead to insulin deficiency and glucose intolerance (hence, the hyperglycemia detected in this patient). Although relatively rare, this cancer represents the fifth most common cause of cancer-related deaths in the U.S., since it comes to medical attention in advanced, usually inoperable, stages. Acute cholangitis (**choice A**) is usually secondary to a gallstone in the common bile duct. In this case, pain and obstructive jaundice are associated with fever, since enteric bacteria gain access to the biliary system and cause infection (ascending cholangitis). Acute cholecystitis (**choice B**) manifests with the classic picture of biliary colic, characterized by pain in the right hypochondrium radiating to the right shoulder. It is usually related to a small gallstone that becomes impacted in the cystic duct, resulting in blockage of bile outflow and acute distention and inflammation of gallbladder. Acute pancreatitis (**choice C**) is an emergency condition manifesting with extremely severe deep abdominal pain and frequently associated with shock. Elevated amylase and lipase levels in the serum support the diagnosis. Adenoma of the liver (**choice D**) is most commonly asymptomatic and may be discovered incidentally. However, severe intraperitoneal hemorrhage due to rupture may constitute a life-threatening presentation. Use of oral contraceptives and anabolic steroids are predisposing conditions. Alcoholic hepatitis (**choice E**) is similar to any other form of acute hepatitis in its clinical manifestations and laboratory findings. Abdominal pain, nausea, mild jaundice, low-grade fever, and malaise are associated with elevated serum transaminase levels. Amebic liver abscess (**choice F**) presents with right upper quadrant pain and fever. Usually, the patient comes to medical attention after several weeks of symptoms. A recent travel history to tropical countries is often elicited. Liver tests are mildly abnormal. Ultrasonography, CT, or MRI demonstrates the location and size of the lesion. Focal nodular hyperplasia (**choice G**) is a nodular lesion consisting of hepatocytes and a stellate-shaped fibrous stroma (a sort of “focal cirrhosis”). It is usually asymptomatic. It is most commonly associated with chronic alcohol abuse. Gilbert syndrome (**choice H**) is a form of hereditary jaundice due to glucuronyl transferase deficiency. Bilirubin is thus mostly of the indirect (unconjugated) type. The condition is benign and asymptomatic (except for jaundice). Hemochromatosis (**choice I**) is an autosomal recessive hereditary disorder caused by excessive intestinal absorption of iron. Iron deposition results in cirrhosis, cardiomyopathy, pancreatic damage with diabetes, and skin hyperpigmentation (due to melanin overproduction, not iron itself). Hepatocellular carcinoma (**choice J**) is most often associated with cirrhosis in industrialized countries. Its development may be unsuspected until the cirrhotic patient manifests rapid deterioration and weight loss. Primary biliary cirrhosis (**choice L**) is characteristic of middle-aged women and is associated with high titers of circulating antimitochondrial autoantibodies. Primary sclerosing cholangitis (**choice M**) is a rare disorder affecting patients with some underlying disease, most commonly ulcerative colitis. It is due to a chronic inflammatory reaction of probable autoimmune origin, which results in sclerosis and obliteration of the extrahepatic bile ducts. Progressively severe obstructive jaundice is the presenting picture. Rotor syndrome (**choice N**) is another hereditary form of jaundice caused by deficient liver excretion of conjugated bilirubin. Since activity of glucuronyl transferase is normal, most circulating bilirubin is of the direct (conjugated) type. The condition is benign. Viral hepatitis (**choice O**) manifests with an acute syndrome characterized by right upper abdominal pain, nausea, malaise, lowgrade fever, and laboratory signs of hepatocellular necrosis. This picture is shared by all forms of viral hepatitis, but acute infection due to HCV is usually asymptomatic.
48
A 10-year-old boy comes to medical attention because of jaundice and right upper abdominal tenderness for 2 days. Laboratory studies show elevated serum aminotransferases, hyperbilirubinemia, and prolonged prothrombin time. A gray-green ring is found in the Descemet membrane of the cornea on slit-lamp examination. The urinary copper level is high. (A) Acute cholangitis (B) Acute cholecystitis (C) Acute pancreatitis (D) Adenoma of the liver (E) Alcoholic hepatitis (F) Amebic liver abscess (G) Focal nodular hyperplasia (H) Gilbert syndrome (I) Hemochromatosis (J) Hepatocellular carcinoma (K) Pancreatic carcinoma (L) Primary biliary cirrhosis (M) Primary sclerosing cholangitis (N) Rotor syndrome (O) Viral hepatitis (P) Wilson disease
**Respuesta: P** The correct **answer is P**. Wilson disease should be suspected in any case of cirrhosis or chronic hepatitis manifesting in children or young adults. The presence of Kayser-Fleischer rings in the cornea is pathognomonic of this condition. Neuropsychiatric manifestations may precede or follow hepatic involvement. Wilson disease is an autosomal recessive disorder caused by mutations of a gene that codes for a copper-transporting protein in the hepatocytes (not ceruloplasmin). The diagnosis is confirmed by low serum ceruloplasmin levels, high urinary copper excretion, and high copper content in liver biopsy. Acute cholangitis (**choice A**) is usually secondary to a gallstone in the common bile duct. In this case, pain and obstructive jaundice are associated with fever, since enteric bacteria gain access to the biliary system and cause infection (ascending cholangitis). Acute cholecystitis (**choice B**) manifests with the classic picture of biliary colic, characterized by pain in the right hypochondrium radiating to the right shoulder. It is usually related to a small gallstone that becomes impacted in the cystic duct, resulting in blockage of bile outflow and acute distention and inflammation of gallbladder. Acute pancreatitis (**choice C**) is an emergency condition manifesting with extremely severe deep abdominal pain and frequently associated with shock. Elevated amylase and lipase levels in the serum support the diagnosis. Adenoma of the liver (**choice D**) is most commonly asymptomatic and may be discovered incidentally. However, severe intraperitoneal hemorrhage due to rupture may constitute a life-threatening presentation. Use of oral contraceptives and anabolic steroids are predisposing conditions. Alcoholic hepatitis (**choice E**) is similar to any other form of acute hepatitis in its clinical manifestations and laboratory findings. Abdominal pain, nausea, mild jaundice, low-grade fever, and malaise are associated with elevated serum transaminase levels. Amebic liver abscess (**choice F**) presents with right upper quadrant pain and fever. Usually, the patient comes to medical attention after several weeks of symptoms. A recent travel history to tropical countries is often elicited. Liver tests are mildly abnormal. Ultrasonography, CT, or MRI demonstrates the location and size of the lesion. Focal nodular hyperplasia (**choice G**) is a nodular lesion consisting of hepatocytes and a stellate-shaped fibrous stroma (a sort of “focal cirrhosis”). It is usually asymptomatic. It is most commonly associated with chronic alcohol abuse. Gilbert syndrome (**choice H**) is a form of hereditary jaundice due to glucuronyl transferase deficiency. Bilirubin is thus mostly of the indirect (unconjugated) type. The condition is benign and asymptomatic (except for jaundice). Hemochromatosis (**choice I**) is an autosomal recessive hereditary disorder caused by excessive intestinal absorption of iron. Iron deposition results in cirrhosis, cardiomyopathy, pancreatic damage with diabetes, and skin hyperpigmentation (due to melanin overproduction, not iron itself). Hepatocellular carcinoma (**choice J**) is most often associated with cirrhosis in industrialized countries. Its development may be unsuspected until the cirrhotic patient manifests rapid deterioration and weight loss. Primary biliary cirrhosis (**choice L**) is characteristic of middle-aged women and is associated with high titers of circulating antimitochondrial autoantibodies. Primary sclerosing cholangitis (**choice M**) is a rare disorder affecting patients with some underlying disease, most commonly ulcerative colitis. It is due to a chronic inflammatory reaction of probable autoimmune origin, which results in sclerosis and obliteration of the extrahepatic bile ducts. Progressively severe obstructive jaundice is the presenting picture. Rotor syndrome (**choice N**) is another hereditary form of jaundice caused by deficient liver excretion of conjugated bilirubin. Since activity of glucuronyl transferase is normal, most circulating bilirubin is of the direct (conjugated) type. The condition is benign. Viral hepatitis (**choice O**) manifests with an acute syndrome characterized by right upper abdominal pain, nausea, malaise, lowgrade fever, and laboratory signs of hepatocellular necrosis. This picture is shared by all forms of viral hepatitis, but acute infection due to HCV is usually asymptomatic.
49
A 66-year-old man on long-term hemodialysis for chronic renal failure presents with bone pains and proximal muscle weakness. The glomerular filtration rate (GFR) is less than 25% of normal. X-ray films reveal generalized osteopenia, particularly pronounced in the phalanges and distal clavicle, and ectopic calcifications in soft tissues around joints. (A) Accumulation of uremic toxins (B) Hyperkalemia (C) Platelet dysfunction (D) Salt and water retention (E) Secondary hyperparathyroidism (F) Shunting of blood through arteriovenous fistula
**Respuesta: E** The correct **answer is E**. When GFR falls below 25%, phosphorus excretion becomes impaired, leading to hyperphosphatemia. Hyperphosphatemia results in hypocalcemia, triggering increased release of parathyroid hormone (PTH; choice E). High PTH levels increase phosphaturia and normalize serum calcium levels, but at the expense of bone calcium. Concomitantly, renal failure results in decreased hydroxylation of 25- hydroxycholecalciferol to 1,25-dihydroxycholecalciferol (the metabolically active form of vitamin D3 ), leading to decreased intestinal absorption of calcium. Loss of renal function causes metabolic acidosis, and the excess hydrogen ions in the blood are buffered by calcium salts deriving from bone. Finally, aluminum, used in chronic renal failure as a phosphorus binder, may itself deposit in the bone, contributing to bone damage. All these factors, in varying degrees, result in renal osteodystrophy, which manifests with rarefaction of bone (osteomalacia) on x-ray films, bone pains, muscle weakness, and pathologic fractures. On x-ray films, the phalanges and the distal third of the clavicles appear more severely affected.
50
A 54-year-old woman with progressive renal failure secondary to diabetes mellitus is found to have a blood pressure of 160/95 mm Hg during a routine follow-up examination. Her blood pressure was previously within normal limits. She is on peritoneal dialysis and dietary management with restriction of protein, water, and salt intake. Treatment with subcutaneous injections of recombinant erythropoietin for anemia was recently started. (A) Accumulation of uremic toxins (B) Hyperkalemia (C) Platelet dysfunction (D) Salt and water retention (E) Secondary hyperparathyroidism (F) Shunting of blood through arteriovenous fistula
**Respuesta: D** The correct **answer is D**. Hypertension is an inevitable consequence, and the most common complication, of chronic renal failure. Control of hypertension is crucial in the management of renal failure as high blood pressure accelerates renal damage. While fluid and salt retention (**choice D**) plays the most important pathogenetic role, other factors may contribute. Recombinant erythropoietin, used to treat anemia, causes hypertension in about 20% of cases. Hypertension may develop abruptly at the beginning of treatment. Accumulation of uremic toxins (**choice A**) has deleterious effects on many tissues and organs. It is thought responsible for the development of uremic encephalopathy (obtunded sensorium), peripheral neuropathy, and uremic pericarditis. The occurrence of any of these complications warrants initiation of renal replacement therapy to eliminate uremic toxins. Hyperkalemia (**choice B**) usually develops when GFR falls below 10% of normal, but many endogenous and exogenous factors may hasten its appearance. Hyperkalemia interferes with neuromuscular junctions and causes muscle weakness, abdominal distention, and diarrhea. It also affects cardiac fiber excitability, resulting in arrhythmias. Chronic hyperkalemia is treated with dietary restriction and, if necessary, oral administration of an ion exchange resin such as sodium polystyrene sulfonate. Platelet dysfunction (**choice C**) is thought to be the principal mechanism of coagulopathy associated with chronic renal failure. While platelet number might be only slightly decreased, platelet adhesiveness and aggregation are impaired, which manifests with prolonged bleeding time. Patients with chronic renal failure, therefore, frequently develop purpura and petechiae. Shunting of blood through arteriovenous fistula (**choice F**) occurs in patients treated with hemodialysis in which vascular access is guaranteed by establishing a permanent arteriovenous fistula. Shunting of blood through such fistula may contribute to the development of congestive heart failure. Other factors include anemia and salt and fluid retention